2008 it e critique

78
American Board of Family Medicine 2008 IN-TRAINING EXAMINATION CRITIQUE BOOK This book contains the answers to each question in the In-Training Examination, as well as a critique which provides a rationale for the correct answer. Bibliographic references are included at the end of each critique to facilitate any further study you may wish to do in a particular area. Copyright© 2008 The American Board of Family Medicine, Inc. All rights reserved.

Upload: rofi-modi

Post on 11-Nov-2014

45 views

Category:

Documents


4 download

TRANSCRIPT

Page 1: 2008 It e Critique

American Board of Family Medicine

2008 IN-TRAINING EXAMINATION

CRITIQUE BOOK

This book contains the answers to each question in the In-Training Examination, as well as a critique whichprovides a rationale for the correct answer. Bibliographic references are included at the end of each critiqueto facilitate any further study you may wish to do in a particular area.

Copyright© 2008 The American Board of Family Medicine, Inc. All rights reserved.

Page 2: 2008 It e Critique

1

Item 1

ANSWER: B

This patient has a clinical presentation consistent with acute sinusitis. Failure to respond to adequateantibiotic therapy suggests either a complication, progression to chronic sinusitis, or a different, confoundingdiagnosis. The diagnostic procedure of choice in this situation is coronal CT of the sinuses, due to itsincreased sensitivity and competitive cost when compared with standard radiographs. Cultures of the nasaldischarge give unreliable results because of bacterial contamination from the resident flora of the nose. Theother options listed do not contribute to the diagnosis and treatment of sinusitis.

Ref: Mandell GL, Bennett JE, Dolin R (eds): Mandell, Douglas, and Bennett’s Principles and Practice of Infectious Diseases, ed6. Churchill Livingstone, 2005, pp 772-782.

Item 2

ANSWER: D

This patient has psychotic depression with suicidal ideations and has not responded to maximum doses ofseveral antidepressants. He is more likely to respond to electroconvulsive therapy than to counseling or achange in medication.

Ref: Lisanby SH: Electroconvulsive therapy for depression. N Engl J Med 2007;359(19):1939-1945.

Item 3

ANSWER: E

Acne neonatorum occurs in up to 20% of newborns. It typically consists of closed comedones on theforehead, nose, and cheeks, and is thought to result from stimulation of sebaceous glands by maternal andinfant androgens. Parents should be counseled that lesions usually resolve spontaneously within 4 monthswithout scarring. Findings in erythema toxicum neonatorum include papules, pustules, and erythema.Candida and herpes lesions usually present with vesiculopustular lesions in the neonatal period. Miliaconsists of 1- to 2-mm pearly keratin plugs without erythema, and may occur on the trunk and limbs.

Ref: O’Connor NR, McLaughlin MR, Ham P: Newborn skin: Part I. Common rashes. Am Fam Physician 2008;77(1):47-52.

Item 4

ANSWER: E

Most women do not regularly perform breast self-examinations, even though it allows motivated women tobe in control of this aspect of their health care. Evidence from large, well-designed, randomized trials ofadequate duration has shown that the performance of regular breast self-examinations by trained women doesnot reduce breast cancer–specific mortality or all-cause mortality. The U.S. Preventive Services Task Forcefound insufficient evidence (an I rating) to recommend breast self-examinations. A Cochrane reviewconcluded that breast self-examinations have no beneficial effect and increase the number of biopsiesperformed.

Ref: Knutson D, Steiner E: Screening for breast cancer: Current recommendations and future directions. Am Fam Physician2007;75(11):1660-1666.

Page 3: 2008 It e Critique

2

Item 5

ANSWER: C

In acute ankle sprains, functional treatment with a semi-rigid brace (Aircast) or a soft lace-up brace isrecommended over immobilization. Casting or posterior splinting is no longer recommended. Elasticbandaging does not offer the same lateral and medial support. External ankle support has been shown toimprove proprioception.

Ref: Gravlee JR, Van Durme DJ: Braces and splints for musculoskeletal conditions. Am Fam Physician 2007;75(3):342-347.

Item 6

ANSWER: E

This patient has classic symptoms of irritable bowel syndrome (IBS) and meets the Rome criteria by having3 days per month of abdominal pain for the past 3 months, a change in the frequency of stool, andimprovement with defecation. According to current clinical guidelines IBS can be diagnosed by history,physical examination, and routine laboratory testing, as long as there are no warning signs. Warning signsinclude rectal bleeding, anemia, weight loss, fever, a family history of colon cancer, onset of symptoms afterage 50, and a major change in symptoms. Colonoscopy, CT, and GI contrast studies are not indicated. Agluten-free diet would not be indicated since the antibody tests for celiac disease are negative. Antidiarrhealagents such as loperamide are generally safe and effective in the management of diarrheal symptoms in IBS.

Ref: Mayer EA: Irritable bowel syndrome. N Engl J Med 2008;358(16):1692-1699.

Item 7

ANSWER: C

Not only has the incidence of gonorrhea increased since 2002, but the rate of quinolone-resistant infectionhas also increased. Ceftriaxone is therefore the currently recommended treatment, and amoxicillin,ciprofloxacin, and erythromycin are no longer recommended because of resistance to these drugs.Doxycycline can be used but should be continued for 7 days.

Ref: Van Vranken M: Prevention and treatment of sexually transmitted diseases: An update. Am Fam Physician2007;76(12):1827-1832.

Item 8

ANSWER: B

Asymptomatic patients in excellent health often present with this characteristic chest radiograph pattern,which is usually due to histoplasmosis infection, especially if the patient has been in the midwestern UnitedStates. Exposure to bird or bat excrement is a common cause, and treatment is usually not needed. Thispattern is not characteristic of the other infections listed, although miliary tuberculosis is a remote possibilityin spite of the negative PPD skin test.

Ref: Fauci AS, Braunwald E, Kasper DL, et al (eds): Harrison’s Principles of Internal Medicine, ed 17. McGraw-Hill, 2008, pp1244-1246.

Page 4: 2008 It e Critique

3

Item 9

ANSWER: D

Nonverbal communication is important for identifying issues that a patient may be hiding or is unwilling todivulge. Some nonverbal clues, however, are culturally biased. Many older or less-educated Mexican-Americans consider direct eye contact to be disrespectful. Because a physician is held in high regard, thesepatients will often either look down or look at another, more “equal” person in the room while beinginterviewed. Many Americans, on the other hand, may consider a lack of eye contact to be negative,implying that a patient is unsure of the information they are providing, has poor self-esteem, or is hidingsomething.

Ref: Purnell LD, Paulanka BJ: Guide to Culturally Competent Health Care. FA Davis Company, 2005, pp 339-351.

Item 10

ANSWER: C

The Advisory Committee on Immunization Practices (ACIP) recommends routine pneumococcal vaccinationfor healthy patients starting at age 65 years. Patients over 2 years of age with various chronic diseases andpatients who are immunocompromised also should be vaccinated (SOR A). Children less than 2 years of ageshould receive the 7-valent conjugate vaccine as a part of their routine well child vaccinations at 2, 4, 6, and12 months of age. There is a lack of data concerning the safety of the vaccine when given three or moretimes.

Ref: Prevention of pneumococcal disease: Recommendations of the Advisory Committee on Immunization Practices (ACIP).MMWR 1997;46(RR-08):1-24. 2) Recommended adult immunization schedule—United States, October 2005–September2006. MMWR 2005;54(40):Q1-Q4.

Item 11

ANSWER: A

Developmental dysplasia of the hip encompasses both subluxation and dislocation of the newborn hip, aswell as anatomic abnormalities. It is more common in firstborns, females, breech presentations,oligohydramnios, and patients with a family history of developmental dysplasia.

Experts are divided with regard to whether hip subluxation can be merely observed during the newbornperiod, but if there is any question of a hip problem on examination by 2 weeks of age, the recommendationis to refer to a specialist for further testing and treatment. Studies show that these problems disappear by 1week of age in 60% of cases, and by 2 months of age in 90% of cases. Triple diapering should not be usedbecause it puts the hip joint in the wrong position and may aggravate the problem. Plain radiographs maybe helpful after 4–6 months of age, but prior to that time the ossification centers are too immature to be seen.

Page 5: 2008 It e Critique

4

Because the condition can be difficult to diagnose, and can result in significant problems, the currentrecommendation is to treat all children with developmental dysplasia of the hip. Closed reduction andimmobilization in a Pavlik harness, with ultrasonography of the hip to ensure proper positioning, is thetreatment of choice until 6 months of age. The American Academy of Pediatrics recommends ultrasoundscreening at 6 weeks for breech girls, breech boys (optional), and girls with a positive family history ofdevelopmental dysplasia of the hip. Other countries have recommended universal screening, but a reviewof the literature has not shown that the benefits of early diagnosis through universal screening outweigh therisks and potential problems of overtreating.

Ref: Storer SK, Skaggs DL: Developmental dysplasia of the hip. Am Fam Physician 2006;74(8):1310-1316. 2) Torchia MM,Phillips W: Developmental dysplasia of the hip. UpToDate, 2007.

Item 12

ANSWER: E

Plantar fasciitis is an overuse injury due to microtrauma of the plantar fascia where it attaches at the medialcalcaneal tubercle. The patient experiences heel or arch pain, which often is worse upon arising and takingthe first few steps of the morning. Examination reveals tenderness at the site and pain with dorsiflexion ofthe toes. Stress fractures often cause pain at rest that intensifies with weight bearing.

Treatment strategies include relative rest, ice, NSAIDs, and prefabricated shoe inserts that provide archsupport, as well as heel cord and plantar fascia stretching. Currently, there is evidence against the use ofextracorporeal shockwave therapy. If conservative therapy fails, a corticosteroid injection may be useful.Surgery is reserved for patients refractory to 6–12 months of uninterrupted conservative therapy.

Ref: Cosca DD, Navazio F: Common problems in endurance athletes. Am Fam Physician 2007;76(2):237-244.

Item 13

ANSWER: E

Methamphetamine dependence is very difficult to treat. No medications have been approved by the FDAfor the treatment of this problem, nor have any studies shown consistent benefit to date. The standardtherapy for methamphetamine dependence is outpatient behavioral therapies, especially with casemanagement included. Therapy must be individualized. Support groups and 12-step drug-treatmentprograms may be helpful.

Ref: Winslow BT, Voorhees KI, Pehl KA: Methamphetamine abuse. Am Fam Physician 2007;76(8):1169-1174.

Item 14

ANSWER: C

This is a classic incidentaloma. Nodules are detected in up to 50% of thyroid sonograms and carry a low riskof malignancy (<5%). If the TSH level is normal, nuclear scanning and further thyroid studies are notnecessary. Nodules smaller than 1 cm are difficult to biopsy and thyroid surgery is not indicated for whatis almost certainly benign disease. It is reasonable to follow small nodules with clinical examinations andperiodic sonograms.

Ref: Hegedus L: The thyroid nodule. N Engl J Med 2004;351(17):1764-1771.

Page 6: 2008 It e Critique

5

Item 15

ANSWER: A

Metformin is widely accepted as the first-line drug for type 2 diabetes mellitus. It is relatively effective, safe,and inexpensive, and has been used widely for many years. Unlike other oral hypoglycemics and insulin,it does not cause weight gain. It should be started at the same time as lifestyle modifications, rather thanwaiting to see if a diet and exercise regimen alone will work.

If metformin is not effective, a sulfonylurea, a thiazolidinedione, or insulin can be added, with the choicebased on the severity of the hyperglycemia.

Ref: Heine RJ, Diamant M, Mbanya JC, et al: Management of hyperglycemia in type 2 diabetes: The end of recurrent failure? BMJ2006;333(7580):1200-1204.

Item 16

ANSWER: C

This patient's history and examination meet the criteria for vascular dementia published by the NationalInstitute of Neurological Disorders and Stroke, and the Association Internationale pour la Neurosciences(NINDS-AIREN). Significant findings include cognitive decline from a previously higher level offunctioning, manifested by impairment of memory and of two or more cognitive domains, and evidence ofcerebrovascular disease by focal signs on neurologic examination, consistent with stroke. To fully meet theNINDS-AIREN criteria, she would need to have neuroimaging that demonstrates characteristic vasculardementia lesions.

Ref: Aggarwal NT, DeCarli C: Vascular dementia: Emerging trends. Semin Neurol 2007;27(1):66-77.

Item 17

ANSWER: E

Nonpharmacologic therapy is recommended for all patients with an overactive bladder. Pelvic floor muscletraining (e.g., Kegel exercises) and bladder training are proven effective in urge incontinence or overactivebladder, as well as in stress and mixed incontinence. In motivated patients, training may be more effectivethan medications such as oxybutynin and newer muscarinic receptor antagonists such as solifenacin.Tamsulosin is used in benign prostatic hypertrophy and phenazopyridine is a urinary tract anesthetic that hasnot been recommended for treating overactive bladder.

Ref: Epstein BJ, Gums JG, Molina E: Newer agents for the management of overactive bladder. Am Fam Physician2006;74(12):2061-2068.

Item 18

ANSWER: A

This patient has galactorrhea, which is defined as a milk-like discharge from the breast in the absence ofpregnancy in a non-breastfeeding patient who is more than 6 months post partum. It is more common inwomen ages 20–35 and in women who are previously parous. It also can occur in men. Medication sideeffect is the most common etiology.

Page 7: 2008 It e Critique

6

The most common pharmacologic cause of galactorrhea is oral contraceptives. Oral contraceptives thatcontain estrogen can both suppress prolactin inhibitory factor and stimulate the pituitary directly, both ofwhich can cause galactorrhea. Other medications that can cause galactorrhea include metoclopramide,cimetidine, risperidone, methyldopa, codeine, morphine, verapamil, SSRIs, butyrophenones,dopamine-receptor blockers, tricyclics, phenothiazines, and thioxanthenes.

Breast cancer is unlikely to present with a bilateral milky discharge. The nipple discharge associated withcancer is usually unilateral and bloody.

Pituitary tumors are a pathologic cause of galactorrhea due to the hyperprolactinemia that is caused by theblockage of dopamine from the hypothalamus, or by the direct production of prolactin. However, patientsoften have symptoms such as headache, visual disturbances, temperature intolerance, seizures, disorderedappetite, polyuria, and polydipsia. Patients with prolactinomas often have associated amenorrhea. Thesetumors are associated with marked levels of serum prolactin, often >200 ng/mL.

Hypothalamic lesions such as craniopharyngioma, primary hypothalamic tumor, metastatic tumor,histiocytosis X, tuberculosis, sarcoidosis, and empty sella syndrome are significant but infrequent causes ofgalactorrhea, and generally cause symptoms similar to those of pituitary tumors, particularly headache andvisual disturbances.

It is rare for primary hypothyroidism to cause galactorrhea in adults. Symptoms that would be a clue to thisdiagnosis include fatigue, constipation, menstrual irregularity, weight changes, and cold intolerance.

Ref: Leung AK, Pacaud D: Diagnosis and management of galactorrhea. Am Fam Physician 2004;70(3):543-550.

Item 19

ANSWER: E

Monotherapy for hypertension in African-American patients is more likely to consist of diuretics or calciumchannel blockers than �-blockers or ACE inhibitors. It has been suggested that hypertension inAfrican-Americans is not as angiotensin II–dependent as it appears to be in Caucasians.

Ref: The Seventh Report of the Joint National Committee on Prevention, Detection, Evaluation, and Treatment of High BloodPressure. National High Blood Pressure Education Program, 2004, NIH Publication No. 04-5230, p 39. 2) Rao S, CherukuriM, Mayo HG: Clinical inquiries. What is the best treatment for hypertension in African-Americans? J Fam Pract2007;56(2):149-151.

Item 20

ANSWER: B

In patients with ischemic chest pain, the EKG is important for determining the need for fibrinolytic therapy.In addition to ST elevation �1 mm in two or more contiguous standard limb leads and �2 mm in two or morecontiguous precordial leads, patients should also have new left bundle branch block. In a patient with apresumed MI, left bundle branch block suggests occlusion of the left anterior descending artery, placing asignificant portion of the left ventricle in jeopardy. Thrombolytic therapy could be harmful in patients withST-segment depression only. Frequent unifocal ventricular ectopy may warrant antiarrhythmic therapy, butnot thrombolytic therapy.

Ref: Marx JA (ed): Rosen’s Emergency Medicine: Concepts and Clinical Practice, ed 6. Mosby Elsevier, 2006, pp 1189-1192.

Page 8: 2008 It e Critique

7

Item 21

ANSWER: B

More than a dozen vaccines are available for diseases with a high prevalence in developing countries. Theprimary care physician should make sure that international travelers are up to date on routine immunizations,given that vaccine-based immunity to tetanus, diphtheria, polio, and measles wanes over time, and that thesediseases are highly prevalent abroad. While location-specific situations may require particularimmunizations such as typhoid, yellow fever, or hepatitis B vaccine, and/or administration of �-globulin,hepatitis A vaccine is recommended for nearly all international travelers.

Ref: Re VL III, Gluckman SJ: Travel immunizations. Am Fam Physician 2004;70(1):89-99.

Item 22

ANSWER: C

The most useful diagnostic tool for evaluating patients with heart failure is two-dimensionalechocardiography with Doppler to assess left ventricular ejection fraction (LVEF), left ventricular size,ventricular compliance, wall thickness, and valve function. The test should be performed during the initialevaluation. Radionuclide ventriculography can be used to assess LVEF and volumes, and MRI or CT alsomay provide information in selected patients. Chest radiography (posteroanterior and lateral) and 12-leadelectrocardiography should be performed in all patients presenting with heart failure, but should not be usedas the primary basis for determining which abnormalities are responsible for the heart failure.

Ref: Smith L: Practice guidelines: Diagnosis and management of chronic heart failure in adults. Am Fam Physician2007;75(5):742-755.

Item 23

ANSWER: A

A positive dipstick for hemoglobin without any RBCs noted in the urine sediment indicates either freehemoglobin or myoglobin in the urine. Since the specimen in this case was a fresh sample, significant RBChemolysis within the urine would not be expected. If a transfusion reaction occurs, haptoglobin binds enoughfree hemoglobin in the serum to give it a pink coloration. Only when haptoglobin is saturated will the freehemoglobin be excreted in the urine. Myoglobin is released when skeletal muscle is destroyed by trauma,infarction, or intrinsic muscle disease. If the hematuria were due to trauma there would be many RBCsvisible on microscopic examination of the urine.

Free hemoglobin resorption from hematomas does not occur. Porphyria may cause urine to be burgundycolored, but it is not associated with a positive urine test for hemoglobin.

Ref: Sauret JM, Marinides G, Wang GK: Rhabdomyolysis. Am Fam Physician 2002;65(5):907-912. 2) Goldman L, Ausiello D(eds): Cecil Medicine, ed 23. Saunders, 2008, pp 798-802.

Page 9: 2008 It e Critique

8

Item 24

ANSWER: E

The patient described has an inflammatory myopathy of the polymyositis/dermatomyositis group. Proximalmuscle involvement and elevation of serum muscle enzymes such as creatine kinase and aldolase arecharacteristic. Corticosteroids are the accepted treatment of choice.

It is extremely unlikely that Duchenne’s muscular dystrophy would present after age 30. In amyotrophiclateral sclerosis, an abnormal neurologic examination with findings of upper motor neuron dysfunction ischaracteristic. Patients with myasthenia gravis characteristically have optic involvement, often presentingas diplopia. The predominant symptom of aseptic necrosis of the femoral head is pain rather than proximalmuscle weakness. Elevated muscle enzymes are not characteristic.

Ref: Fauci AS, Braunwald E, Kasper DL, et al (eds): Harrison’s Principles of Internal Medicine, ed 17. McGraw-Hill, 2008, pp2696-2703.

Item 25

ANSWER: C

Gilbert syndrome is an autosomal dominant disease characterized by indirect hyperbilirubinemia caused byimpaired glucuronyl transferase activity. The workup includes studies to exclude hemolysis (CBC,reticulocyte count, and haptoglobin) and liver disease (AST, ALT, alkaline phosphatase, and prothrombintime). Alcoholic liver disease is associated with a greater elevation of AST than of ALT. Dubin-Johhnsonsyndrome is a benign liver disease distinguished by direct or conjugated hyperbilirubinemia. Imaging studiesare not required to confirm Gilbert syndrome; such studies are more useful for conditions involvingconjugated hyperbilirubinemia. Other causes of indirect hyperbilirubinemia include hematoma, infection,cardiac disease, rhabdomyolysis, living at high altitude, thyrotoxicosis, and some medications.

Ref: Fauci AS, Braunwald E, Kasper DL, et al (eds): Harrison’s Principles of Internal Medicine, ed 17. McGraw-Hill, 2008, pp1928-1930.

Item 26

ANSWER: D

This patient is experiencing primary dysmenorrhea, a common finding in adolescents, with estimates ofprevalence ranging from 20% to 90%. Because symptoms started at a rather young age and she has pain onlyduring menses, endometriosis or other significant pelvic pathology is unlikely. An infection is doubtful,considering that she is not sexually active and that symptoms have been present for 2 years. In the absenceof red flags, a pelvic examination, laboratory evaluation, and pelvic ultrasonography are not necessary at thistime. However, they can be ordered if she does not respond to simple treatment.

NSAIDs such as naproxen have a slight effect on platelet function, but because they inhibit prostaglandinsynthesis they actually decrease the volume of menstrual flow and lessen the discomfort of pelvic cramping.Acetaminophen would have no effect on prostaglandins.

Ref: French L: Dysmenorrhea. Am Fam Physician 2005;71(2):285-291.

Page 10: 2008 It e Critique

9

Item 27

ANSWER: B

Although breast milk is the ideal source of nutrition for healthy term infants, supplementation with 200IU/day of vitamin D is recommended beginning at 2 months of age and continuing until the child isconsuming at least 500 mL/day of formula or milk containing vitamin D (SOR B). The purpose ofsupplementation is to prevent rickets. Unless the baby is anemic or has other deficiencies, neither iron nora multivitamin is necessary. Parents often mistakenly think babies need additional water, which can beharmful because it decreases milk intake and can cause electrolyte disturbances. Cereal should not be starteduntil 4 months of age.

Ref: Burke BL, Hall RW: Newborn care: 12 beliefs that shape practice (but should they?). J Fam Pract 2007;56(10):802-807.

Item 28

ANSWER: A

A Cochrane meta-analysis of nicotine replacement therapy (NRT) found that it almost doubles a smoker’schances of quitting (SOR A). There was no benefit to tapering NRT as compared to abrupt discontinuation.Treatment for 8 weeks was as effective as a longer course. No one type of NRT is significantly moreeffective, but combining several types may aid a relapsed smoker in his or her next quit attempt.

Ref: Coffay AO: Smoking cessation: Tactics that make a big difference. J Fam Pract 2007;56(10):817-824.

Item 29

ANSWER: A

Delayed gastric emptying may be caused or exacerbated by medications for diabetes, including amylinanalogues (e.g., pramlintide) and glucagon-like peptide 1 (e.g., exenatide). Delayed gastric emptying has adirect effect on glucose metabolism, in addition to being a means of reducing the severity of postprandialhyperglycemia. In a clinical trial of exenatide, nausea occurred in 57% of patients and vomiting occurredin 19%, which led to the cessation of treatment in about one-third of patients. The other medications listeddo not cause delayed gastric emptying.

Ref: Camilleri M: Diabetic gastroparesis. N Engl J Med 2007;356(8):820-829.

Item 30

ANSWER: A

Clozapine was the first atypical antipsychotic drug, so designated because it has antipsychotic effects withoutthe adverse effects on movement seen with first-generation agents, in addition to having enhanced therapeuticefficacy compared with first-generation drugs. Because of these advantages, it was introduced into clinicalpractice in the United States despite a serious known adverse effect: an increased incidence ofagranulocytosis. Although only clozapine causes agranulocytosis in a substantial proportion of patients,many second-generation drugs produce clinically significant weight gain.

Page 11: 2008 It e Critique

10

Ref: Freedman R: Schizophrenia. N Engl J Med 2003;349(18):1738-1749.

Item 31

ANSWER: E

Long-standing ulcerative colitis (UC) is associated with an increased risk of colon cancer. The greater theduration and anatomic extent of involvement, the greater the risk. Initial colonoscopy for patients withpancolitis of 8–10 years’ duration (regardless of the patient’s age) should be followed up with surveillanceexaminations every 1–2 years, even if the disease is in remission.

All of the other options listed are features typically associated with Crohn’s disease. Virtually all patientswith UC have rectal involvement, even if that is the only area affected. In Crohn’s disease, rectalinvolvement is variable. Noncontinuous and transmural inflammation are also more common with Crohn’sdisease. Transmural inflammation can lead to eventual fistula formation, which is not seen in UC.

Ref: Langan RC, Gotsch PB, Krafczyk MA, et al: Ulcerative colitis: Diagnosis and treatment. Am Fam Physician2007;76(9):1323-1330, 1331.

Item 32

ANSWER: C

This patient most likely has periodontitis of the tooth’s roots with cellulitis, complicated by an apical abscess.This infection is caused by anaerobic oral bacteria. Clindamycin or amoxicillin/clavulanate is preferred forantibiotic treatment. Doxycycline, trimethoprim/sulfamethoxazole, ciprofloxacin, and cephalexin havelimited effectiveness against anaerobes and would not be indicated.

Ref: Nguyen DH, Martin JT: Common dental infections in the primary care setting. Am Fam Physician 2008;77(6):797-802.

Item 33

ANSWER: D

Exercise-induced bronchoconstriction (EIB) is a very common and underdiagnosed condition in athletes.

1It is defined as a 10% lowering of FEV when challenged with exercise. The exercise required to causebronchoconstriction is 5–8 minutes at 80% of maximal oxygen consumption.

EIB is much more common in high-ventilation sports, such as track and cross-country skiing. It is also morecommon in winter sports, because of the inspiration of cold, dry air. In some studies the incidence amongcross-country skiers is as high as 50%, and 40% of those who have positive tests for bronchospasm areunaware of the problem. A physical examination, as well as pulmonary function tests at rest and before andafter bronchodilators, will be normal unless there is underlying asthma. Among athletes with EIB, 10% willnot have asthma.

Bronchoprovocative testing can be ordered, but if it is not available a trial with an albuterol inhaler isreasonable. Cardiomyopathy or valvular dysfunction not found during the physical examination is possible,but much less likely. Psychological stresses are also a possible etiology, but should not receive undueattention, especially when simple questioning is not productive and more likely diagnoses have not beenruled out. Poor training methods are also possible, but in a competitive athlete this is not the most likelycause.

Page 12: 2008 It e Critique

11

Ref: Parsons JP, Mastronarde JG: Exercise-induced bronchoconstriction in athletes. Chest 2005;128(6):3966-3974.

Item 34

ANSWER: A

A number of useful strategies for promoting adherence to a chronic medical regimen can be employed inchildren. Adding flavors to unpleasant tasting medicines is helpful (SOR B). Chocolate flavoring isespecially useful for masking the taste of bitter medications. Using medications that are given only once ortwice a day is associated with compliance rates of greater than 70% (SOR B). Consistent advice given bymultiple members of the health-care team reinforces the importance of following a medication regimen.Parental use of rewards for children who take their medicine properly helps improve adherence. Involvingchildren in decisions concerning their care gives them a sense of control and improves adherence.

Other strategies for improving adherence include patient handouts, keeping financial costs in mind whenprescribing, advising patients to incorporate dosing into daily routines such as meals, keeping tally sheets,and using visual reminders such as notes on the refrigerator.

Ref: Gardiner P, Dvorkin L: Promoting medication adherence in children. Am Fam Physician 2006;74(5):793-798, 800.

Item 35

ANSWER: A

Amantadine and rimantadine are not recommended for the treatment of influenza A because of thedevelopment of resistance to these drugs. Resistance is not a problem with neuramidase inhibitors such asoseltamivir in immunocompetent patients. Zanamivir is not recommended for treatment of children underthe age of 7. Although this child has recently received influenza vaccine, this is not a contraindication todrug therapy.

Ref: Antiviral drugs for influenza. Med Lett Drugs Ther 2007;49(1272):85-86.

Item 36

ANSWER: B

Several studies have demonstrated that SSRIs are safe and effective in treating depression in patients withcoronary disease, particularly those with a history of previous episodes of depression. Medications haveperformed significantly better than intensive interpersonal psychotherapy in this setting. Electroconvulsivetherapy is not considered first-line therapy in the absence of severe symptoms. While it may be effective forsleep disturbance, amitriptyline has potential cardiac side effects and is unlikely to be effective for thetreatment of depression in low doses.

Ref: Glassman AH, O’Connor CM, Califf RM, et al: Sertraline treatment of major depression in patients with acute MI or unstableangina. JAMA 2002;288(6):701-709. 2) Lesperance F, Frasure-Smith N, Koszycki D, et al: Effects of citalopram andinterpersonal psychotherapy on depression in patients with coronary artery disease. JAMA 2007;297(4):367-379.

Page 13: 2008 It e Critique

12

Item 37

ANSWER: A

Amenorrhea is an indicator of inadequate calorie intake, which may be related to either reduced foodconsumption or increased energy use. This is not a normal response to training, and may be the firstindication of a potential developing problem. Young athletes may develop a combination of conditions,including eating disorders, amenorrhea, and osteoporosis (the female athlete triad). Amenorrhea usuallyresponds to increased calorie intake or a decrease in exercise intensity. It is not necessary for patients suchas this one to stop running entirely, however.

Ref: Master-Hunter T, Heiman DL: Amenorrhea: Evaluation and treatment. Am Fam Physician 2006;73(8):1374-1382.

Item 38

ANSWER: E

This patient has moderate to severe COPD. Smoking cessation is the single most important therapeuticintervention in patients with this condition and should be a priority of care. No existing medications havebeen shown to modify the long-term decline in lung function that is typical of COPD, but smoking cessationdoes prevent this decline. Long-term use of oxygen in COPD patients who also have chronic, severe hypoxia(<88% saturation) can improve quality of life and prolong survival; however, oxygen cannot prevent furtherdecline in lung function. Long-term use of oral corticosteroids is discouraged because of an unfavorablerisk-to-benefit ratio.

Ref: Global strategy for the diagnosis, management, and prevention of chronic obstructive pulmonary disease. MedicalCommunication Resources, 2006, pp 8-15. 2) Rabe KF, Beghe B, Luppi F, et al: Update in chronic obstructive pulmonarydisease 2006. Am J Respir Crit Care Med 2007;175(12):1222-1232.

Item 39

ANSWER: E

Preeclampsia affects as many as 5% of first pregnancies and is manifested as hypertension, proteinuria,edema, and rapid weight gain after 20 weeks gestation. Very young mothers and those over age 35 have ahigher risk. Patients who have had preeclampsia have a fourfold increased risk of hypertension and a twofoldincreased risk of ischemic heart disease, stroke, and venous thromboembolism. There does not appear to bean association between preeclampsia and cancer, breast cancer in particular.

Ref: Bellamy L, Casas JP, Hingorani AD, et al: Pre-eclampsia and risk of cardiovascular disease and cancer in later life: Systematicreview and meta-analysis. BMJ 2007;335(7627):974.

Item 40

ANSWER: D

The differential diagnosis of urinary retention in the elderly is broad. While most causes are benign andreadily treated, the physician must be vigilant in looking for conditions that require urgent intervention.

Page 14: 2008 It e Critique

13

This patient presents with many possible causes of urinary retention, with the most common being benignprostatic hyperplasia. Acute prostatitis, especially in a male with an enlarged prostate, is another relativelycommon reason for obstructive symptoms. This patient’s physical examination and abnormal urinalysissupport this diagnosis, but his normal vital signs and lack of fever suggest he can be treated with an oralfluroquinolone and does not require hospital admission for intravenous therapy.

Medications such as oral decongestants can contribute to urinary retention in men with enlarged prostateglands, and should be used with caution and discontinued if obstructive symptoms occur. Obstipation withstool impaction is another relatively common reason for urinary retention in the elderly and can be treatedwith manual disimpaction and enemas.

In this patient, the presence of increasing low back pain and leg weakness, and the findings of anal sphincterlaxity and numbness in the perianal area on examination, suggest the presence of a serious neurologicetiology such as cauda equina syndrome. Urgent diagnosis and treatment are necessary to reduce morbidity,and MRI should be performed immediately.

The presence of a mildly elevated post-void residual is not an indication for urgent decompression with aFoley catheter.

Ref: Selius BA, Subedi R: Urinary retention in adults: Diagnosis and initial management. Am Fam Physician 2008;77(5):643-650.

Item 41

ANSWER: A

Although JNC-7 guidelines recommend a diuretic as the initial pharmacologic agent for most patients withhypertension, the presence of “compelling indications” may indicate the need for treatment withantihypertensive agents that demonstrate a particular benefit in primary or secondary prevention. JNC-7guidelines recommend ACE inhibitors (ACEIs) or angiotensin receptor blockers (ARBs) for hypertensivepatients with chronic kidney disease (SOR A).

First-line therapy for proteinuric kidney disease includes an ACEI or an ARB. Because these drugs can causeelevations in creatinine and potassium, these levels should be monitored. A serum creatinine level as muchas 35% above baseline is acceptable in patients taking these agents and is not a reason to withhold treatmentunless hyperkalemia develops. If an ACEI or an ARB does not control the hypertension, the addition of adiuretic or a calcium channel blocker may be required. The combination of ACEIs and diuretics may be usedto control hypertension in patients with diabetes mellitus, heart failure, or high coronary disease risk, as wellas post myocardial infarction. Calcium channel blockers are recommended for managing hypertension inpatients with diabetes or high coronary disease risk.

�-Blockers are useful as part of combination therapy in patients with hypertension and heart failure, or postmyocardial infarction.

Ref: The Seventh Report of the Joint National Committee on Prevention, Detection, Evaluation, and Treatment of High BloodPressure. National High Blood Pressure Education Program, 2004, NIH Publication No. 04-5230, pp 37-38. 2) Frank J:Managing hypertension using combination therapy. Am Fam Physician 2008;77(9):1279-1286, 1289.

Page 15: 2008 It e Critique

14

Item 42

ANSWER: D

Although oral antibiotics are overwhelmingly prescribed as initial treatment in acute sinusitis, it has beenshown that the majority of acute illnesses are viral in origin and that 98% of cases will resolve spontaneously.Analgesics are considered the mainstay of therapy for acute sinusitis, according to evidence-basedrecommendations (SOR A). Other treatments should be considered if symptoms are prolonged (>7 days) orsevere (two or more localizing symptoms or signs of serious bacterial complications).

There is little evidence of effectiveness for antihistamines, oral decongestants, or vasoconstrictor sprays.There is also little evidence of effectiveness for nasal lavage in acute sinusitis, although it has an emergingrole in chronic sinusitis.

Ref: Scheid DC, Hamm RM: Acute bacterial rhinosinusitis in adults: Part II. Treatment. Am Fam Physician2004;70(9):1697-1704. 2) Ah-See KW, Evans AS: Sinusitis and its management. BMJ 2007;334(7589):358-361.

Item 43

ANSWER: A

Most tinea infections respond to topical therapy, but oral therapy is required for tinea capitis so that the drugwill penetrate the hair shafts (SOR B). Tinea corporis may require oral therapy in severe cases, but usuallyresponds to topical therapy (SOR A). Oral therapy has a higher likelihood of side effects.

Erythrasma and mycosis fungoides are not fungal diseases.

Ref: Andrews MD, Burns M: Common tinea infections in children. Am Fam Physician 2008;77(10):1415-1420.

Item 44

ANSWER: C

The specificity of a test for a disease is the proportion or percentage of those without the disease who havea negative test. In this case, option A is the sensitivity, i.e., the proportion of those with the disease who havea positive test. Option B is the false-negative rate and option D is the false-positive rate. Option E is theratio of false-negative tests to false-positive tests, a meaningless ratio. The predictive values of positive andnegative tests are extremely important characteristics of a screening test. Determination of these valuesrequires knowledge of the prevalence of the disease in the population screened, as well as the sensitivity,specificity, and false-positive and false-negative rates. Since the prevalence of most diseases is low, thepercentage of those with a positive test (the predictive value of a positive test) is relatively low, even whensensitivity and specificity are high. When prevalence is low, however, the predictive value of a negative testis very high and may approach 100%.

Ref: Fauci AS, Braunwald E, Kasper DL, et al (eds): Harrison’s Principles of Internal Medicine, ed 17. McGraw-Hill, 2008, pp18-19.

Page 16: 2008 It e Critique

15

Item 45

ANSWER: E

Entrapment of the posterior tibial nerve or its branches as the nerve courses behind the medial malleolusresults in a neuritis known as tarsal tunnel syndrome. Causes of compression within the tarsal tunnel includevarices of the posterior tibial vein, tenosynovitis of the flexor tendon, structural alteration of the tunnelsecondary to trauma, and direct compression of the nerve. Pronation of the foot causes pain and paresthesiasin the medial aspect of the ankle and heel, and sometimes the plantar surface of the foot.

The usual site for a stress fracture is the shaft of the second, third, or fourth metatarsals. A herniated nucleuspulposus would produce reflex and sensory changes. Plantar fasciitis is the most common cause of heel painin runners and often presents with pain at the beginning of the workout. The pain decreases during runningonly to recur afterward. Diabetic neuropathy is usually bilateral and often produces paresthesias and burningat night, with absent or decreased deep tendon reflexes.

Ref: Brukner P, Khan K: Clinical Sports Medicine, ed 3. McGraw Hill, 2006, pp 636-637.

Item 46

ANSWER: D

Constitutional growth delay, defined as delayed but eventually normal growth in an adolescent, is usuallygenetic. If evaluation of the short adolescent male reveals no evidence of chronic disease, if his sexualmaturity rating is 2 or 3, and if his height is appropriate for skeletal age he can be told without endocrinologictesting that he will begin to grow taller within a year or so. Adult height may be below average, but cannotbe predicted reliably. Average sexual maturity ratings for a male of 14.3 years are 4 for genitalia and 3 to4 for pubic hair. The history and physical examination would have given clues to any illnesses or nutritionalproblems.

Ref: Kliegman RM, Behrman RE, Jenson HB, et al (eds): Nelson Textbook of Pediatrics, ed 18. Saunders, 2007, p 2297.

Item 47

ANSWER: E

The physiologic changes that accompany aging result in altered pharmacokinetics. Drug distribution is oneimportant factor. In older persons, there is a relative increase in body fat and a relative decrease in lean bodymass, which causes increased distribution of fat-soluble drugs such as diazepam. This also increases theelimination half-life of such medications. The volume of distribution of water-soluble compounds such asdigoxin is decreased in older patients, which means a smaller dose is required to reach a given target plasmaconcentration. There is a predictable reduction in glomerular filtration rate and tubular secretion with aging,which causes decreased clearance of medications in the geriatric population. The absorption of drugschanges little with advancing age. All of these changes are important factors in choosing dosages ofmedications in the elderly population.

Ref: Duthie EH Jr, Katz PR, Malone ML (eds): Practice of Geriatrics, ed 4. Saunders Elsevier, 2007, pp 18-20.

Page 17: 2008 It e Critique

16

Item 48

ANSWER: C

The FDA has established a fetal risk summary dividing drugs into categories. Category A drugs have beenshown in controlled studies to pose no risk. At present there are no category A antibiotics. Most fall intocategories B and C, with category B drugs thought to be relatively safe in pregnancy. When possible, acategory B antibiotic should be chosen for treatment of a pregnant patient. Category C drugs have unknownfetal risk with no adequate human studies, and the possibility of risks and benefits must be considered beforeprescribing them for pregnant women. Category D drugs show some evidence for fetal risk; although theremay be times when use of these drugs is necessary, they should not be used unless there is a very serious orlife-threatening situation. Category X drugs have proven fetal risk and are contraindicated in pregnancy.

Of the drugs listed, only nitrofurantoin is in category B. The others are all category C drugs.

The FDA is currently in the process of revising their classification and labeling for drugs in pregnancy andlactation.

Ref: Briggs GG, Freeman RK, Yaffe SJ (eds): Drugs in Pregnancy and Lactation: A Reference Guide to Fetal and Neonatal Risk.Lippincott Williams & Wilkins, 2005.

Item 49

ANSWER: A

Diplopia, along with other neurologic symptoms such as weakness or difficulty with speech, suggests acentral cause of vertigo and requires a complete workup. Dizziness on first arising, dizziness with rollingover in bed, and dizziness with nausea and vomiting are consistent with peripheral causes of vertigo, suchas benign positional vertigo. Dizziness that occurs after a couple of minutes of hyperventilation suggests apsychogenic cause.

Ref: Labuguen RH: Initial evaluation of vertigo. Am Fam Physician 2006;73(2):244-251. 2) Fauci AS, Braunwald E, Kasper DL,et al (eds): Harrison’s Principles of Internal Medicine, ed 17. McGraw-Hill, 2008, pp 144-146.

Item 50

ANSWER: A

Claudication is exercise-induced lower-extremity pain that is caused by ischemia and relieved by rest. Itaffects 10% of persons over 70 years of age. However, up to 90% of patients with peripheral vasculardisease are asymptomatic.

Initial treatment should consist of vigorous risk factor modification and exercise. Patients who follow anexercise regimen can increase their walking time by 150%. A supervised program may produce betterresults. Risk factors include diabetes mellitus, hypertension, smoking, and hyperlipidemia. Unconventionaltreatments such as chelation have not been shown to be effective. Vasodilating agents are of no benefit.There is no evidence that anticoagulants such as aspirin have a role in the treatment of claudication.

Page 18: 2008 It e Critique

17

Ref: Cassar K: Intermittent claudication. BMJ 2006;333(7576):1002-1005. 2) Sontheimer DL: Peripheral vascular disease:Diagnosis and treatment. Am Fam Physician 2006;73(11):1971-1976.

Item 51

ANSWER: C

Xerosis is a pathologic dryness of the skin that is especially prominent in the elderly. It is probably causedby minor abnormalities in maturation of the epidermis that lead to decreased hydration of the superficialportion of the stratum corneum. Xerosis often intensifies in winter, because of the lower humidity and coldtemperatures.

Stasis dermatitis, due to chronic venous insufficiency, appears as a reddish-brown discoloration of the lowerleg. Lichen simplex chronicus, the end result of habitual scratching or rubbing, usually presents as isolatedhyperpigmented, edematous lesions, which become scaly and thickened in the center. Rosacea is most oftenseen on the face as an erythematous, acneiform eruption, which flushes easily and is surrounded bytelangiectasia. Candidiasis is an opportunistic infection favoring areas that are warm, moist, and macerated,such as the perianal and inguinal folds, inframammary folds, axillae, interdigital areas, and corners of themouth.

Ref: Duthie EH Jr, Katz PR, Malone ML (eds): Practice of Geriatrics, ed 4. Saunders Elsevier, 2007, pp 536-537.

Item 52

ANSWER: B

The most reliable predictor of survival in breast cancer is the stage at the time of diagnosis. Tumor size andlymph node involvement are the main factors to take into account. Other prognostic parameters (tumorgrade, histologic type, and lymphatic or blood vessel involvement) have been proposed as importantvariables, but most microscopic findings other than lymph node involvement correlate poorly with prognosis.Estrogen receptor (ER) status may also predict survival, with ER-positive tumors appearing to be lessaggressive than ER-negative tumors.

Ref: Abeloff MD, Armitage JO, Niederhuber JE, et al (eds): Clinical Oncology, ed 3. Elsevier Churchill Livingstone, 2004, pp2399-2401.

Item 53

ANSWER: E

This patient had a classic simple febrile seizure and no additional diagnostic studies are recommended. Alumbar puncture following a seizure is not routinely recommended in a child over 18 months of age, sinceby that age a patient with meningitis would be expected to demonstrate meningeal signs and symptoms orclinical findings suggesting an intracranial infection. There is no evidence to suggest that routine blood testsor neuroimaging studies are useful in a patient following a first simple febrile seizure, and it has not beenshown that electroencephalography performed either at the time of presentation or within the followingmonth will predict the likelihood of recurrence.

Ref: Provisional Committee on Quality Improvement, Subcommittee on Febrile Seizures: Practice parameter: The neurodiagnosticevaluation of the child with a first simple febrile seizure. Pediatrics 1996;97(5):769-772. 2) Kliegman RM, Behrman RE,Jenson HB, et al (eds): Nelson Textbook of Pediatrics, ed 18. Saunders, 2007, pp 2458-2459.

Page 19: 2008 It e Critique

18

Item 54

ANSWER: E

Osteoarthritis causes changes predominantly in the proximal interphalangeal (PIP) and distal interphalangeal(DIP) joints of the hands, and the carpometacarpal joints of the thumbs. While rheumatoid arthritiscommonly causes subluxations in the metacarpophalangeal joints, this patient’s subluxation is most likelydue to osteoarthritis. The other choices are less likely to cause this problem.

Ref: Klippel JH, Stone JH, Crofford LJ, et al (eds): Primer on the Rheumatic Diseases, ed 13. Springer, 2008, pp 224-228.

Item 55

ANSWER: B

According to the American Diabetes Association guidelines, thiazolidinediones (TZDs) are associated withfluid retention, and their use can be complicated by the development of heart failure. Caution is necessarywhen prescribing TZDs in patients with known heart failure or other heart diseases, those with preexistingedema, and those on concurrent insulin therapy (SOR C).

Older patients can be treated with the same drug regimens as younger patients, but special care is requiredwhen prescribing and monitoring drug therapy. Metformin is often contraindicated because of renalinsufficiency or heart failure. Sulfonylureas and other insulin secretagogues can cause hypoglycemia.Insulin can also cause hypoglycemia, and injecting it requires good visual and motor skills and cognitiveability on the part of the patient or a caregiver. TZDs should not be used in patients with New York HeartAssociation class III or IV heart failure.

Ref: American Diabetes Association: Standards of medical care in diabetes—2007. Diabetes Care 2007;30(Suppl 1):S4-S41.

Item 56

ANSWER: E

Patients with thoracic aneurysms often present without symptoms. With dissecting aneurysms, however, thepresenting symptom depends on the location of the aneurysm. Aneurysms can compress or distort nearbystructures, resulting in branch vessel compression or embolization of peripheral arteries from a thrombuswithin the aneurysm. Leakage of the aneurysm will cause pain, and rupture can occur with catastrophicresults, including severe pain, hypotension, shock, and death. Aneurysms in the ascending aorta may presentwith acute heart failure brought about by aortic regurgitation from aortic root dilatation and distortion of theannulus. Other presenting findings may include hoarseness, myocardial ischemia, paralysis of ahemidiaphragm, wheezing, coughing, hemoptysis, dyspnea, dysphagia, or superior vena cava syndrome.

This diagnosis should be suspected in individuals in their sixties and seventies with the same risk factors asthose for coronary artery disease, particularly smokers. A chest radiograph may show widening of themediastinum, enlargement of the aortic knob, or tracheal displacement. Transesophageal echocardiographycan be very useful when dissection is suspected. CT with intravenous contrast is very accurate for showingthe size, extent of disease, pressure of leakage, and nearby pathology. Angiography is the preferred methodfor evaluation and is best for evaluation of branch vessel pathology. MR angiography provides noninvasivemultiplanar image reconstruction, but does have limited availability and lower resolution than traditionalcontrast angiography.

Page 20: 2008 It e Critique

19

Acute dissection of the ascending aorta is a surgical emergency, but dissections confined to the descendingaorta are managed medically unless the patient demonstrates progression or continued hemorrhage into theretroperitoneal space or pleura. Initial management should reduce the systolic blood pressure to 100–120mm Hg or to the lowest level tolerated.

The use of a �-blocker such as propranolol or labetalol to get the heart rate below 60 beats/min should befirst-line therapy. If the systolic blood pressure remains over 100 mm Hg, intravenous nitroprusside shouldbe added. Vasodilation will induce reflex activation of the sympathetic nervous system, causing increasedventricular contraction and increased shear stress on the aorta.

For descending dissections, surgery is indicated only for complications such as occlusion of a major aorticbranch, continued extension or expansion of the dissection, or rupture (which may be manifested bypersistent or recurrent pain).

Ref: Libby P, Bonow RO, Mann DL, et al (eds): Braunwald’s Heart Disease: A Textbook of Cardiovascular Medicine, ed 8.Saunders, 2008, pp 1469-1486.

Item 57

ANSWER: D

There are several benign murmurs of childhood that have no association with physiologic or anatomicabnormalities. Of these, Still’s murmur best fits the murmur described. The cause of Still’s murmur isunknown, but it may be due to vibrations in the chordae tendinae, semilunar valves, or ventricular wall. Avenous hum consists of a continuous low-pitched murmur caused by collapse of the jugular veins and theirsubsequent fluttering, and it worsens with inspiration or diastole. The murmur of physiologic peripheralpulmonic stenosis (PPPS) is caused by physiologic changes in the newborn’s pulmonary vessels. PPPS isa systolic murmur heard loudest in the axillae bilaterally that usually disappears by 9 months of age. Mitralstenosis causes a diastolic murmur, and Eisenmenger’s syndrome involves multiple abnormalities of the heartthat cause significant signs and symptoms, including shortness of breath, cyanosis, and organomegaly, whichshould become apparent from a routine history and examination.

Ref: Rudolph CD, Rudolph AM (eds): Rudolph’s Pediatrics, ed 21. McGraw-Hill, 2003, pp 1755-1757.

Item 58

ANSWER: B

Severe hyperkalemia (>6.0 mEq/L) requires aggressive treatment. Calcium gluconate has no effect on theplasma potassium level, but it should be given first, as it rapidly stabilizes the membranes of cardiacmyocytes, reducing the risk of cardiac dysrhythmias. Therapies that translocate potassium from the serumto the intracellular space should be instituted next, as they can quickly (albeit temporarily) lower the plasmaconcentration of potassium. These interventions include sodium bicarbonate, glucose with insulin, andalbuterol. Total body potassium can be lowered with sodium polystyrene sulfonate, but this takes longer toaffect the plasma potassium level than translocation methods. In the most severe cases, acute hemodialysiscan be instituted.

Ref: Sood MM, Sood AR, Richardson R: Emergency management and commonly encountered outpatient scenarios in patients withhyperkalemia. Mayo Clin Proc 2007;82(12):1553-1561.

Page 21: 2008 It e Critique

20

Item 59

ANSWER: D

HIV screening is recommended as part of routine prenatal care, even in low-risk pregnancies. Counselingabout cystic fibrosis carrier testing is recommended, but not routine testing. Hepatitis C and parvovirusantibodies are not part of routine prenatal screening. Routine screening for bacterial vaginosis with a vaginalsmear for clue cells is not recommended.

Ref: Human Immunodeficiency Virus Infection, Topic Page. US Preventive Services Task Force, Agency for Healthcare Researchand Quality, 2007. Available at http://www.ahrq.gov/clinic/uspstf/uspshivi.htm

Item 60

ANSWER: B

Postoperative pulmonary complications are important contributors to the risks associated with surgery andanesthesia. Significant postoperative pulmonary complications include atelectasis, pneumonia, respiratoryfailure, and exacerbation of underlying chronic lung disease.

Risk factors for postoperative pulmonary complications can be divided into patient-related factors,procedure-related factors, and laboratory factors. Patient-related risk factors include advanced age, AmericanSociety of Anesthesiologists Physical Status classification of 2 or higher, functional dependence, COPD, andheart failure. Of these, advanced age confers the greatest risk, with risk beginning to increase after age 50.One study found an odds ratio of 5.63 for those age 80 and above.

Procedure-related risk factors include aortic aneurysm repair, nonresective thoracic surgery, abdominalsurgery, neurosurgery, emergency surgery, general anesthesia, head and neck surgery, vascular surgery, andprolonged surgery.

The only laboratory predictor supported by good evidence is a serum albumin level <30 g/L.

Ref: Eagle KA, Smetana GW: What the new guidelines offer for preoperative risk reduction. Patient Care Nurs Pract 2006;21-22.2) Smetana GW, Lawrence VA, Cornell JE, et al: Preoperative pulmonary risk stratification for noncardiothoracic surgery:Systematic review for the American College of Physicians. Ann Intern Med 2006;144(8):581-595.

Item 61

ANSWER: A

Lithium therapy can elevate calcium levels by elevating parathyroid hormone secretion from the thyroidgland. This duplicates the laboratory findings seen with mild primary hyperparathyroidism. If possible,lithium should be discontinued for 3 months before reevaluation (SOR C). This is most important foravoiding unnecessary parathyroid surgery.

Vitamin D and calcium supplementation could contribute to hypercalcemia in rare instances, but they wouldnot cause elevation of parathyroid hormone. Raloxifene has actually been shown to mildly reduce elevatedcalcium levels, and furosemide is used with saline infusions to lower significantly elevated calcium levels.

Page 22: 2008 It e Critique

21

Ref: Bilezikian JP, Silverberg SJ: Asymptomatic primary hyperparathyroidism. N Engl J Med 2004;350(17):1746-1751. 2)Javorsky BR, Dalkin AC: Hypercalcemia: A practical approach to a surprising condition. Emerg Med 2007;(Sept):13-20. 3)Kronenberg HM, Melmed S, Polonsky KS, et al (eds): Williams Textbook of Endocrinology, ed 11. Saunders Elsevier, 2008,pp 392-393.

Item 62

ANSWER: B

Macrolides are considered first-line therapy for Bordetella pertussis infection.Trimethoprim/sulfamethoxazole is considered second-line therapy.

Ref: Wenzel RP, Fowler AA: Acute bronchitis. N Engl J Med 2006;355(20):2125-2130.

Item 63

ANSWER: D

Neglect is the most common form of child abuse (60% of cases) and is the most common cause of death inabused children. It is defined by the Office on Child Abuse and Neglect as failure to provide for a child’sbasic physical, emotional, educational/cognitive, or medical needs.

Ref: McDonald KC: Child abuse: Approach and management. Am Fam Physician 2007;75(2):221-228.

Item 64

ANSWER: A

Any child younger than 29 days old with a fever and any child who appears toxic, regardless of age, shouldundergo a complete sepsis workup and be admitted to the hospital for observation until culture results areknown or the source of the fever is found and treated (SOR A).

Observation only, with close follow-up, is recommended for nontoxic infants 3–36 months of age with atemperature <39.0°C (102.2°F) (SOR B). Children 29–90 days old who appear to be nontoxic and havenegative screening laboratory studies, including a CBC and urinalysis, can be sent home with precautionsand with follow-up in 24 hours (SOR B). Testing for neonatal herpes simplex virus infection should beconsidered in patients with risk factors, including maternal infection at the time of delivery, use of fetal scalpelectrodes, vaginal delivery, cerebrospinal fluid pleocytosis, or herpetic lesions. Testing also should beconsidered when a child does not respond to antibiotics (SOR C).

Ref: Sur DK, Bukont EL: Evaluating fever of unidentifiable source in young children. Am Fam Physician 2007;75(12):1805-1811.

Item 65

ANSWER: B

Acute interstitial nephritis (AIN) is often drug-induced. Discontinuation of medications that are likely tocause AIN is the most important first step in management. If these medications are withdrawn early, mostpatients can be expected to recover normal renal function. Of the medications listed, ibuprofen is the mostlikely offending agent, because all NSAIDs are known to be associated with AIN. Development of AINusually becomes evident approximately 2 weeks after starting a medication and is not dose-related. Othermedications strongly associated with AIN include various antibiotics (particularly cephalosporins,penicillins, sulfonamides, aminoglycosides, and rifampin), diuretics, and miscellaneous medications such

Page 23: 2008 It e Critique

22

as allopurinol.

Ref: Kodner CM, Kudrimoti A: Diagnosis and management of acute interstitial nephritis. Am Fam Physician2003;67(12):2527-2534.

Item 66

ANSWER: E

There are no absolute contraindications to electroconvulsive therapy (ECT), but more complications are seenin patients with a history of recent cerebral hemorrhage, stroke, or increased intracranial pressure. Theefficacy of ECT may be reduced in patients who have not responded to oral antidepressants.

Ref: Lisanby SH: Electroconvulsive therapy for depression. N Engl J Med 2007;357(19):1939-1945.

Item 67

ANSWER: A

Pulmonary emboli, anastomotic leaks, and respiratory failure are responsible for 80% of deaths in the 30 daysfollowing bariatric surgery, with death from pulmonary embolism being the most frequent cause. Woundinfections and marginal ulcers are not uncommon complications of this type of surgery.

Ref: Virgi A, Murr MM: Caring for patients after bariatric surgery. Am Fam Physician 2006;73(8):1403-1408.

Item 68

ANSWER: A

Anemia of chronic disease is characterized by the underproduction of red cells, due to hypoferremia causedby the uptake of iron by the reticuloendothelial system. Total-body iron stores are increased but the iron instorage is not available for red cell production. This anemia is normochromic and normocytic, and isassociated with a reduction in iron, transferrin, and transferrin saturation. Ferritin is either normal orincreased, reflecting both the increased iron within the reticuloendothelial system and increases due toimmune activation (acute phase reactant). In iron deficiency anemia, total-body iron levels are low, leadingto hypochromia and microcytosis, low iron levels, increased transferrin levels, and reduced ferritin levels.This patient’s anemia is most likely multifactorial, with anemia of chronic disease and drug effects playinga role. However, she also has iron deficiency, and searching for a source of blood loss would be important.With thalassemia, marked microcytosis is seen, and with hemolysis of any standing, slight macrocytosis andan increased reticulocyte count would be expected.

Ref: Weiss G, Goodnough LT: Anemia of chronic disease. N Engl J Med 2005;352(10):1011-1023.

Page 24: 2008 It e Critique

23

Item 69

ANSWER: B

Routine use of over-the-counter antihistamines should be discouraged because they are only minimallyeffective in inducing sleep, may reduce sleep quality, and can cause residual drowsiness.

Cognitive-behavioral therapy helps change incorrect beliefs and attitudes about sleep (e.g., unrealisticexpectations, misconceptions, amplifying consequences of sleeplessness). Techniques include reattributiontraining (goal setting and planning coping responses), decatastrophizing (balancing anxious automaticthoughts), reappraisal, and attention shifting. Cognitive-behavioral therapy is recommended as an effective,nonpharmacologic treatment for chronic insomnia (SOR A).

Many herbs and dietary supplements have been promoted as sleep aids. However, with the exceptions ofmelatonin and valerian, there is insufficient evidence of benefit.

Alcohol acts directly on GABA-gated channels, reducing sleep-onset latency, but it increases wakefulnessafter sleep onset and suppresses rapid eye movement (REM) sleep. It also has the potential for abuse andshould not be used as a sleep aid.

Moderate-intensity exercise can improve sleep, but exercising just before bedtime can delay sleep onset.

Ref: Ramakrishnan K, Scheid DC: Treatment options for insomnia. Am Fam Physician 2007;76(4):517-526.

Item 70

ANSWER: B

Many patients have mild hyperparathyroidism that becomes evident only with an added calcium load.Thiazide diuretics reduce calcium excretion and can cause overt symptoms in a patient whosehyperparathyroidism would otherwise have remained asymptomatic. The finding of a normal parathyroidhormone (PTH) level in a patient with hypercalcemia is diagnostic for hyperparathyroidism, since PTHshould be suppressed in the presence of elevated calcium. Symptomatic hypercalcemia causes dehydrationbecause of both intestinal symptoms and diuresis. Reversible renal insufficiency can result, and can becomepermanent if it is long-standing and severe.

Conversely, renal failure usually causes hypocalcemia, but can cause hypercalcemia resulting from tertiaryhyperparathyroidism. This develops after severe hyperphosphatemia and vitamin D deficiency eventuallyproduce hypersecretion of PTH. This patient’s renal insufficiency is not severe enough to cause tertiaryhyperparathyroidism. Milk alkali syndrome is hypercalcemia resulting from a chronic overdose of calciumcarbonate, and is becoming more common as more patients take calcium and vitamin D supplements. In milkalkali syndrome, and other causes of hypercalcemia such as sarcoidosis, the PTH level is appropriatelysuppressed.

Ref: Carroll MF, Schade DS: A practical approach to hypercalcemia. Am Fam Physician 2003;67(9):1959-1966. 2) Wermers RA,Keams AE, Jenkins GD, et al: Incidence and clinical spectrum of thiazide-associated hypercalcemia. Am J Med2007;120(10):e9-e15.

Page 25: 2008 It e Critique

24

Item 71

ANSWER: C

A therapeutic exercise program will reduce both pain and disability in patients with osteoarthritis of the knee(SOR A). There is no evidence to support the use of capsaicin cream, but NSAIDs will reduce pain and thereare proven therapies that will improve function of the patient’s knee. While intra-articular corticosteroidsare effective in relieving pain in the short term (up to 4 weeks), there is no evidence for long-term efficacy.There is not good evidence to support the use of glucosamine for treating osteoarthritis of the knee. Onesystematic review found it no more effective than placebo.

Ref: Scott D, Kowalcyzk A: Osteoarthritis of the knee. BMJ Clin Evid 2007;08:1121.

Item 72

ANSWER: A

Pulmonary function tests are usually classified as normal, compatible with a restrictive defect, or consistent

1with obstructive airway disease. In restrictive ventilatory processes, the FVC is decreased, the FEV is

1decreased or normal, and the absolute FEV /FVC is >0.7. In obstructive airway problems, findings include

1 1a normal or decreased FVC, a decreased FEV , and an absolute FEV /FVC <0.7. When simple spirometrysuggests a restrictive ventilatory problem, the patient should undergo full pulmonary function testing forstatic lung volume measurements and diffusing capacity of the lung for carbon monoxide. If spirometrysuggests an obstructive problem, it should be repeated after administering an inhaled bronchodilator.Because this patient has a history of pulmonary tuberculosis, a TB skin test is inappropriate and would notprovide any useful information.

Ref: Barreiro TJ, Perillo I: An approach to interpreting spirometry. Am Fam Physician 2004;69(5):1107-1114.

Item 73

ANSWER: D

Temporomandibular joint (TMJ) disorders occur in a large number of adults. The etiology is varied, butincludes dental malocclusion, bruxism (teeth grinding), anxiety, stress disorders, and, rarely, rheumatoidarthritis. Dental occlusion problems, once thought to be the primary etiology, are not more common inpersons with TMJ disorder. While dental splints have been commonly recommended, the evidence for andagainst their use is insufficient to make a recommendation either way. Physical therapy modalities such asiontophoresis or phonophoresis may benefit some patients, but there is no clearly preferred treatment.Radiologic imaging is unnecessary in the vast majority of patients, and should therefore be reserved forchronic or severe cases. In fact, the majority of patients with TMJ disorders have spontaneous resolutionof symptoms, so noninvasive symptomatic treatments and “tincture of time” are the best approach for most.

Ref: Buescher JJ: Temporomandibular joint disorders. Am Fam Physician 2007;76(10):1477-1482.

Page 26: 2008 It e Critique

25

Item 74

ANSWER: A

This patient has cutaneous larva migrans, a common condition caused by dog and cat hookworms. Fecalmatter deposited on soil or sand may contain hookworm eggs that hatch and release larvae, which areinfective if they penetrate the skin. Walking barefoot on contaminated ground can lead to infection.

Echinococcosis (hydatid disease) is caused by the cestodes (tapeworms) Echinococcus granulosus andEchinococcus multilocularis, found in dogs and other canids. It infects humans who ingest eggs that are shedin the animal’s feces and results in slow-growing cysts in the liver or lungs, and occasionally in the brain,bones, or heart. Toxoplasmosis is caused by the protozoa Toxoplasma gondii, found in cat feces. Humanscan contract it from litter boxes or feces-contaminated soil, or by consuming infected undercooked meat.It can be asymptomatic, or it may cause cervical lymphadenopathy, a mononucleosis-like illness; it can alsolead to a serious congenital infection if the mother is infected during pregnancy, especially during the firsttrimester. Toxocariasis due to Toxocara canis and Toxocara cati causes visceral or ocular larva migrans inchildren who ingest soil contaminated with animal feces that contains parasite eggs, often found in areas suchas playgrounds and sandboxes.

Ref: Rabinowitz PM, Gordon Z, Odofin L: Pet-related infections. Am Fam Physician 2007;76(9):1314-1322.

Item 75

ANSWER: A

This patient has been clinically stable despite losing what appears to be a fair amount of blood into hispleural space after fracturing two ribs, a condition referred to as hemothorax. The treatment of choice in thiscondition is to remove the bloody fluid and re-expand the associated lung. This therapy is felt to decreaseany ongoing blood loss by having the lung pleura put a direct barrier over the site that is bleeding. It alsoprevents the development of empyema or fibrosis, which could occur if the blood were to remain.

Ref: Feliciano DV, Mattox KL, Moore EE, et al: Trauma, ed 6. McGraw-Hill, 2007, chap 26.

Item 76

ANSWER: E

The etiology of fibromyalgia remains unknown, but it is a common condition that is underdiagnosed. It ismuch more common in women than in men; additional risk factors include being divorced, having a lowincome level, and not completing high school. The pathogenesis may be related to central sensitization anddysregulation of the hypothalamic (pituitary) adrenal axis.

Commonly associated symptoms include headache (often migraine in type), anxiety, depression, anddizziness. The diagnosis is specific, and requires the finding of at least 11 tender points in 18 possiblespecific anatomic locations, with the pain occurring both above and below the waist on both sides of the bodyfor at least 3 months.

Page 27: 2008 It e Critique

26

There is strong evidence for the effectiveness of both pharmacologic therapies such as cyclobenzaprine andnonpharmacologic therapies such as aerobic exercise, cognitive-behavioral therapy, and multidisciplinaryapproaches that include patient education and exercise. The evidence of effectiveness is weak forchiropractic therapy, electrotherapy, massage therapy, and ultrasound. There is no known benefit fromcorticosteroids, opioids, thyroid hormone, NSAIDs, melatonin, flexibility exercises, or trigger-pointinjections.

Ref: Chakrabarty S, Zoorob R: Fibromyalgia. Am Fam Physician 2007;76(2):247-254.

Item 77

ANSWER: A

This child had transient tachypnea of the newborn, the most common cause of neonatal respiratory distress.It is a benign condition due to residual pulmonary fluid remaining in the lungs after delivery. Risk factorsinclude cesarean delivery, macrosomia, male gender, and maternal asthma and/or diabetes mellitus. Theother conditions listed cause neonatal respiratory distress, but do not resolve spontaneously. They also causeadditional significant abnormal findings on physical examination and/or ancillary studies such as imagingand laboratory studies.

Ref: Hermansen CL, Lorah KN: Respiratory distress in the newborn. Am Fam Physician 2007;76(7):987-994.

Item 78

ANSWER: A

The patient described initially showed signs of acute mountain sickness. These include headache in anunacclimatized person who recently arrived at an elevation >2500 m (8200 ft), plus the presence of one ormore of the following: anorexia, nausea, vomiting, insomnia, dizziness, or fatigue. The patient’s conditionthen deteriorated to high-altitude cerebral edema, defined as the onset of ataxia and/or altered consciousnessin someone with acute mountain sickness. The management of choice is a combination of descent andsupplemental oxygen. Often, a descent of only 500–1000 m (1600–3300 ft) will lead to resolution of acutemountain sickness. Simulated descent with a portable hyperbaric chamber also is effective, but descentshould not be delayed while awaiting helicopter delivery. If descent and/or administration of oxygen is notpossible, medical therapy with dexamethasone and/or acetazolamide may reduce the severity of symptoms.Nifedipine has also been shown to be helpful in cases of high-altitude pulmonary edema where descentand/or supplemental oxygen is unavailable.

Ref: Hackett PH, Roach RC: High-altitude illness. N Engl J Med 2001;345(2)107-114. 2) Clarke C: Acute mountain sickness:Medical problems associated with acute and subacute exposure to hypobaric hypoxia. Postgrad Med J 2006;82(973):748-753.

Item 79

ANSWER: B

Overt hyperthyroidism causes an increase in neonatal morbidity from preterm birth and low birth weight.Propylthiouracil should be considered the treatment of choice because methimazole may be associated withcongenital anomalies. I is contraindicated in pregnancy because of radiation dangers to the fetus, as well131

as thyroid destruction. Although subtotal thyroidectomy is a viable treatment option, it is recommended onlyif medical therapy is unsuccessful.

Page 28: 2008 It e Critique

27

Ref: Reid JR, Wheeler SF: Hyperthyroidism: Diagnosis and treatment. Am Fam Physician 2005;72(4):623-630. 2) Managementof thyroid dysfunction during pregnancy and postpartum: An Endocrine Society clinical practice guideline. The EndocrineSociety, 2007.

Item 80

ANSWER: A

Permission for the release of patient information should always be in writing. Although the actual medicalrecord is the property of the physician, the information in the chart is the property of the patient. Ethicallyand legally, patients have a right to the information in their medical records, and it cannot be withheld fromthe patient or a third party (at the request of the patient), even if medical bills are unpaid or the physician isconcerned about the patient.

Ref: Snyder L, Leffler C: American College of Physicians Ethics and Human Rights Committee: Ethics manual: Fifth edition. AnnIntern Med 2005;142(7):560-582.

Item 81

ANSWER: D

Patellofemoral pain syndrome is a clinical diagnosis and is the most common cause of knee pain in theoutpatient setting. It is characterized by anterior knee pain, particularly with activities that overload the joint,such as stair climbing, running, and squatting. Patients complain of “popping,” “catching,” “stiffness,” and“giving way.” On examination there will be a positive “J” sign, with the patella moving from a medial toa lateral location when the knee is fully extended from the 90° position. This is caused by an imbalance inthe medial and lateral forces acting on the patella. “Locking” is not characteristic of patellofemoral painsyndrome, so loose bodies or a meniscal tear should be considered if this is found.

Ref: Dixit S, DiFiori JP, Burton M, et al: Management of patellofemoral pain syndrome. Am Fam Physician 2007;75(2):194-202.

Item 82

ANSWER: C

The current (2004) recommendation of the U.S. Preventive Services Task Force is that children over the ageof 6 months receive oral fluoride supplementation if the primary drinking water source is deficient influoride. They cite “fair evidence” (B recommendation) that such supplementation reduces the incidenceof dental caries and conclude that the overall benefit outweighs the potential harm from dental fluorosis.

Dental fluorosis is chiefly a cosmetic staining of the teeth, is uncommon with currently recommended doses,and has no other functional or physiologic consequences. Fluoridated toothpaste can cause fluorosis inchildren younger than 2 years of age, and is therefore not recommended in this age group. By itself it doesnot reliably prevent tooth decay.

Fluoride varnish, applied by a dental or medical professional, is another treatment option to prevent caries.It provides longer lasting protection than fluoride rinses, but since it is less concentrated, it may carry a lowerrisk of fluorosis than other forms of supplementation.

Page 29: 2008 It e Critique

28

Oral fluoride supplementation for children over the age of 6 months is based not only on age but on theconcentration of fluoride in the primary source of drinking water, whether it be tap water or bottled water.Most municipal water supplies in the U.S. are adequately fluoridated, but concentrations vary. Fluorideconcentrations in bottled water vary widely. If the concentration is greater than 0.6 ppm no supplementationis needed, and if given, may result in fluorosis. Lower concentrations of fluoride may indicate the need forpartial or full-dose supplementation.

Ref: Recommendations for using fluoride to prevent and control dental caries in the United States. MMWR 2001;50(RR-14):1-42.2) US Preventive Services Task Force: Prevention of dental caries in preschool children: Recommendations and rationale.Am Fam Physician 2004;70(8):1529-1532.

Item 83

ANSWER: C

�-Blockers are recommended to reduce mortality in symptomatic patients with heart failure (SOR A).Because polypharmacy can reduce compliance, the role that digoxin will ultimately play in heart failure isunclear. The Digitalis Investigation Group study revealed a trend toward increased mortality among womenwith heart failure who were taking digoxin, but digoxin levels were higher among women than men.

There is no evidence that warfarin decreases mortality in patients with heart failure. There is also no evidencethat amiodarone decreases mortality from heart failure in patients with no history of atrial fibrillation.

Calcium channel blockers should be used with caution in patients with heart failure because they can causeperipheral vasodilation, decreased heart rate, decreased cardiac contractility, and decreased cardiacconduction.

Ref: Chavey WE, Bleske BE, Van Harrison R, et al: Pharmacologic management of heart failure caused by systolic dysfunction.Am Fam Physician 2008;77(7):957-964. 2) Horowitz BZ: Toxicity, calcium channel blocker. eMedicine 2007. Available athttp://www.emedicine.com/emerg/topic75.htm.

Item 84

ANSWER: E

An acute rupture of any major tendon should be repaired as soon as possible. Acute tears of the rotator cuffshould be repaired within 6 weeks of the injury if possible (SOR C). Nonsurgical management is notrecommended for active persons. Observing for an extended period will likely lead to retraction of thedetached tendon, possible resorption of tissue, and muscle atrophy.

Ref: Matsen FA III: Clinical practice. Rotator-cuff failure. N Engl J Med 2008;358(20):2138-2147.

Item 85

ANSWER: A

A hydrocele of the tunica vaginalis testis occurs frequently at birth but usually resolves in a few weeks ormonths. No treatment is indicated during the first year of life unless there is a clinically evident hernia. Asimple scrotal hydrocele without communication with the peritoneal cavity and no associated hernia shouldbe excised if it has not spontaneously resolved by the age of 12 months. Aspirating the mass for diagnosticor therapeutic reasons is not recommended, since a loop of bowel may be injured; removing the fluid isineffective.

Page 30: 2008 It e Critique

29

Ref: Kliegman RM, Behrman RE, Jenson HB, et al (eds): Nelson Textbook of Pediatrics, ed 18. Saunders, 2007, p 2264.

Item 86

ANSWER: E

Physical examination of the abdomen is often unreliable for detecting significant intra-abdominal injury,especially in the head-injured or intoxicated patient. In a hemodynamically unstable patient with a high-riskmechanism of injury and altered mental status, peritoneal lavage is the quickest, most reliable modality todetermine whether there is a concomitant intra-abdominal injury requiring laparotomy. CT of the abdomenand contrast duodenography may complement lavage in stable patients with negative or equivocal lavageresults, but in an unstable or uncooperative patient these studies are too time-consuming or require ill-advisedsedation. Ultrasonography may also complement lavage in selected patients, but its usefulness is limited inthe acute situation. MRI is extremely accurate for the anatomic definition of structural injury, but logisticslimit its practical application in acute abdominal trauma.

Ref: Tintinalli JE, Kelen GD, Stapczynski JS (eds): Emergency Medicine: A Comprehensive Study Guide, ed 6. McGraw-Hill,2004, pp 1615-1616. 2) Townsend CM Jr, Beauchamp RD, Evers BM, et al: Sabiston Textbook of Surgery: The BiologicalBasis of Modern Surgical Practice, ed 18. Saunders, 2008, pp 502-504.

Item 87

ANSWER: B

Health-care workers exposed to a patient with meningococcal meningitis are at increased risk of developingsystemic disease and should receive chemoprophylaxis, especially if the contact is intimate. Secondary casesusually occur within 4 days of the initial case. Therefore, prophylactic treatment should begin as soon aspossible. Rifampin has been shown to be 90% effective in eliminating meningococcus from the nasopharynx.Other appropriate chemoprophylactic agents include minocycline and ciprofloxacin.

Even high doses of penicillin may not eradicate nasopharyngeal meningococci. Prednisone has no place inchemoprophylaxis. Meningococcal vaccine appears to have clinical efficacy, but it usually takes more than5 days to become effective.

Ref: Goldman L, Ausiello D (eds): Cecil Medicine, ed 23. Saunders, 2008, pp 2215-2216.

Item 88

ANSWER: C

The finding of a red, raised, friable lesion on the cervix, or a well-demarcated cervical lesion, mandates abiopsy to exclude cervical carcinoma, and treatment for chronic cervicitis should not be started until thebiopsy results are available. A Papanicolaou test by itself is insufficient if there is a grossly visible lesion,as false-negatives occur in 10%–50% of tests.

Ref: Berek JS (ed): Berek & Novak’s Gynecology, ed 14. Lippincott Williams & Wilkins, 2007, p 20.

Page 31: 2008 It e Critique

30

Item 89

ANSWER: D

Domestic violence can affect children, intimate partners, and older adults. It is a serious medical problemthat should be considered in the care of patients and families. There is great variation in the profiles ofpatients affected by domestic violence. Neither demographic factors nor psychological problems have beenfound to be consistent predictors of victimization or violence. Domestic violence cuts across all racial,socioeconomic, religious, and ethnic lines. The only consistent risk factor for being a victim of domesticviolence is female gender.

Ref: Neufeld B: SAFE questions: Overcoming barriers to the detection of domestic violence. Am Fam Physician1996;53(8):2575-2580. 2) Screening for Family and Intimate Partner Violence, Topic Page. US Preventive Services TaskForce, Agency for Healthcare Research and Quality, 2004. Available at http://www.ahrq.gov/clinic/uspstf/uspsfamv.htm.

Item 90

ANSWER: E

Randomized, placebo-controlled trials have shown that isolated systolic hypertension in the elderly respondsbest to diuretics, and to a lesser extent �-blockers. Diuretics are preferred, although long-actingdihydropyridine calcium channel blockers may also be used. In the case described, �-blockers or clonidinemay worsen the depression. Thiazide diuretics may also improve osteoporosis, and would be the mostcost-effective and useful agent in this instance.

Ref: The Seventh Report of the Joint National Committee on Prevention, Detection, Evaluation, and Treatment of High BloodPressure. National High Blood Pressure Education Program, 2004, NIH Publication No. 04-5230, pp 29-30, 44-46.

Item 91

ANSWER: A

The first study ordered in any patient with suspected subarachnoid hemorrhage should be a head CT withoutcontrast. It will reveal subarachnoid hemorrhage in 100% of cases within 12 hours of the bleed, and it isuseful for identifying other sources for the headache, for predicting the site of the aneurysm, and forpredicting cerebral vasospasm and poor outcome. As blood is cleared from the affected area, CT sensitivitydrops to 93% within 24 hours, and to 50% at 7 days. Patients with a positive CT result for subarachnoidhemorrhage should proceed directly to angiography and treatment. Patients with a suspected subarachnoidhemorrhage who have negative or equivocal results on head CT should have a lumbar puncture. MRI andCT with contrast are not used for the diagnosis of acute subarachnoid hemorrhage.

Ref: Suarez JI, Tarr RW, Selman WR: Aneurysmal subarachnoid hemorrhage. N Engl J Med 2006;354(4):387-396.

Item 92

ANSWER: A

Effective oral antiviral drugs are now available for chronic hepatitis B (at a cost of about $20 per day) andcan be added to highly active antiretrovirals. The recent trend in the treatment of newly diagnosed patientswith AIDS and hepatitis B is to treat both problems initially, selecting AIDS drugs that are also active againsthepatitis B. Various agents to stimulate appetite are used in declining HIV patients, but have little benefit.Liver transplantation has been done in a few cases of coinfection with hepatitis B and HIV, but the hepatitis

Page 32: 2008 It e Critique

31

B viremia has to be suppressed first.

Ref: Lin KW, Kirchner JT: Hepatitis B. Am Fam Physician 2004;69(1):75-82.

Item 93

ANSWER: B

Secondary osteoporosis can result from a variety of endocrine, nutritional, or genetic disorders, as well asfrom prolonged use of certain medications. Anticonvulsants such as phenytoin increase the hepaticmetabolism of vitamin D, thereby reducing intestinal calcium absorption. Other medications that adverselyaffect bone mineral density include glucocorticoids, cyclosporine, phenobarbital, and heparin. Thiazidediuretics reduce urinary calcium loss and are believed to preserve bone density with long-term use.Benzodiazepines and SSRIs have not been associated with increases in bone loss or in hip fractures.Raloxifene, a selective estrogen receptor modulator, is indicated for the prevention and treatment ofosteoporosis in postmenopausal women.

Ref: Kronenberg H, Melmed S, Polonsky K, et al: Williams Textbook of Endocrinology, ed 11. Saunders, 2008, p 1288.

Item 94

ANSWER: D

Meckel’s diverticulum is the most common congenital abnormality of the small intestine. It is prone tobleeding because it may contain heterotopic gastric mucosa. Abdominal pain, distention, and vomiting maydevelop if obstruction has occurred, and the presentation may mimic appendicitis.

Children with appendicitis have right lower quadrant pain, abdominal tenderness, guarding, and vomiting,but not rectal bleeding.

With acute viral gastroenteritis, vomiting usually precedes diarrhea (usually without blood) by several hours,and abdominal pain is typically mild and nonfocal with no localized tenderness. The incidence of midgutvolvulus peaks during the first month of life, but it can present anytime in childhood. Volvulus may presentin one of three ways: as a sudden onset of bilious vomiting and abdominal pain in the neonate; as a historyof “feeding problems” with bilious vomiting that now appears to be due to bowel obstruction; or, lesscommonly, as a failure to thrive with severe feeding intolerance. Necrotizing enterocolitis is typically seenin the neonatal intensive-care unit, occurring in premature infants in their first few weeks of life. The infantsare ill, and signs and symptoms include lethargy, irritability, decreased oral intake, abdominal distention, andbloody stools. A plain abdominal film showing pneumatosis intestinalis, caused by gas in the intestinal wall,is diagnostic of this disease.

Ref: McCollough M, Sharieff GQ: Abdominal pain in children. Pediatr Clin North Am 2006;53(1):107-137.

Page 33: 2008 It e Critique

32

Item 95

ANSWER: E

If supraventricular tachycardia is refractory to adenosine or rapidly recurs, the tachycardia can usually beterminated by the administration of intravenous verapamil or a �-blocker. If that fails, intravenouspropafenone or flecainide may be necessary. It is also important to look for and treat possible contributingcauses such as hypovolemia, hypoxia, or electrolyte disturbances. Electrical cardioversion may be necessaryif these measures fail to terminate the tachyarrhythmia.

Ref: Delacretaz E: Supraventricular tachycardia. N Engl J Med 2006;354(10):1039-1051.

Item 96

ANSWER: A

Hormone replacement therapy that includes estrogen has been shown to decrease osteoporosis and bonefracture risk. The risk for colorectal cancer also is reduced after 5 years of estrogen use. The risk formyocardial infarction, stroke, breast cancer, and venous thromboembolism increases with long-term use.

Ref: Farquhar CM, Marjoribanks J, Lethaby A, et al: Long-term hormone therapy for perimenopausal and postmenopausal women.Cochrane Database Syst Rev 2005;(3):CD004143. 2) Magee SR, Taylor JS: Cochrane for clinicians: Hormone therapy inpostmenopausal and perimenopausal women. Am Fam Physician 2006;74(9):1501-1502.

Item 97

ANSWER: B

Most children will be evaluated for a febrile illness before 36 months of age, with the majority having aself-limited viral illness. Nontoxic-appearing febrile infants 29–90 days of age who have a negativescreening laboratory workup, including a CBC with differential and a normal urinalysis, can be sent homeand followed up in 24 hours (SOR B). A second option is to obtain blood cultures and stool studies, or achest film if indicated by the history or examination, and spinal fluid studies if empiric antibiotics are to begiven. This infant’s clinical status did not indicate that any of these additional studies should be performed,and empiric antibiotic treatment is not planned.

Observation with no follow-up is an appropriate strategy in nontoxic children, but only if the child is 3–36months of age and the temperature is under 39°C (SOR B). Nontoxic children 3–36 months of age shouldbe reevaluated in 24–48 hours if the temperature is over 39°C. Although a positive response to antipyreticshas been considered an indication of a lower risk of serious bacterial infection, there is no correlationbetween fever reduction and the likelihood of such an infection.

Any infant younger than 29 days, and any infant or child with a toxic appearance regardless of age, shouldundergo a complete sepsis workup and be admitted for observation until culture results are obtained or thesource of the fever is found and treated (SOR A).

Ref: Sur DK, Bukont EL: Evaluating fever of unidentifiable source in young children. Am Fam Physician 2007;75(12):1805-1811.

Page 34: 2008 It e Critique

33

Item 98

ANSWER: D

3 4With subclinical thyroid dysfunction, TSH is either below or above the normal range, free T or T levels arenormal, and the patient has no symptoms of thyroid disease. Subclinical hypothyroidism (TSH >10 �U/mL)is likely to progress to overt hypothyroidism, and is associated with increased LDL cholesterol. Subclinicalhyperthyroidism (TSH <0.1 �U/mL) is associated with the development of atrial fibrillation, decreased bonedensity, and cardiac dysfunction. Neither type of subclinical thyroid dysfunction is associated with diabetesmellitus. There is insufficient evidence of benefit to warrant early treatment of either condition.

Ref: Wilson GR, Curry RW Jr: Subclinical thyroid disease. Am Fam Physician 2005;72(8):1517-1524.

Item 99

ANSWER: D

Patients with chronic illness, diabetes mellitus, cerebrospinal fluid leaks, chronic bronchopulmonarydysplasia, cyanotic congenital heart disease, or cochlear implants should receive one dose of pneumococcalpolysaccharide vaccine after 2 years of age, and at least 2 months after the last dose of pneumococcalconjugate vaccine. Revaccination with polysaccharide vaccine is not recommended for these patients.Individuals with sickle cell disease, those with anatomic or functional asplenia, immunocompromised personswith renal failure or leukemia, and HIV-infected persons should receive polysaccharide vaccine on thisschedule and should be revaccinated at least 3 years after the first dose.

Ref: Zimmerman RK, Middleton DB: Vaccines for persons at high risk, 2007. J Fam Pract 2007;56(2 Suppl Vaccines):S38-S46,C4-C5.

Item 100

ANSWER: A

Polymyalgia rheumatica is a disease of the middle-aged and elderly. Discomfort is common in the neck,shoulders, and hip girdle areas. There is an absence of objective joint swelling, and findings tend to besymmetric. Characteristically, the erythrocyte sedimentation rate and C-reactive protein levels aresignificantly elevated; however, these tests are nonspecific. Occasionally there are mild elevations of liverenzymes, but muscle enzymes, including creatine kinase, are not elevated in this disorder. Elevation ofmuscle enzymes strongly suggests another diagnosis.

Polymyositis and dermatomyositis are associated with variable levels of muscle enzyme elevations duringthe active phases of the disease.

Drug-induced myopathies such as those seen with the statin family of cholesterol-lowering medications tendto produce some elevation of muscle enzymes during the course of the disorder.

Hypothyroidism is associated with creatine kinase elevation. It should be strongly considered in the patientwith unexplained, otherwise asymptomatic creatine kinase elevation found on a routine chemistry profile.Hyperthyroidism may cause muscle disease and loss of muscle, but it is not associated with creatine kinaseelevation.

Page 35: 2008 It e Critique

34

Ref: Wallach J: Interpretation of Diagnostic Tests, ed 5. Little Brown & Co, 1992, pp 240, 245, 259. 2) Rendt K: Inflammatorymyopathies: Narrowing the differential diagnosis. Cleve Clin J Med 2001;68(6):505-519. 3) Dale DC, Federman DD (eds):ACP Medicine, 2004-2005 ed. American College of Physicians, 2004, pp 1348-1391. 4) Dambro MR (ed): Griffith’s5-Minute Clinical Consult, 2005. Lippincott Williams & Wilkins, 2004, p 872. 5) Needham M, Mastaglia FL: Inclusion bodymyositis: Current pathogenetic concepts and diagnostic and therapeutic approaches. Lancet Neurol 2007;6(7):620-631.

Item 101

ANSWER: E

The current Dietary Reference Intake (DRI) recommendation for vitamin D is 200 IU/day for all womenbetween the ages of 9 and 50 years; pregnancy or lactation does not affect the recommendation. The DRIdoubles to 400 IU daily for women age 51–70 and triples to 600 IU daily for women over the age of 70. Themaximum daily oral intake of vitamin D thought to be safe is 2000 IU/day for all females over the age of 12months.

Ref: Dietary reference intakes: Recommended intakes for individuals, vitamins. National Academy of Sciences, 2004.

Item 102

ANSWER: A

Eradication of Helicobacter pylori significantly reduces the risk of ulcer recurrence and rebleeding inpatients with duodenal ulcer, and reduces the risk of peptic ulcer development in patients on chronic NSAIDtherapy. Eradication has minimal or no effect on the symptoms of nonulcer dyspepsia and gastroesophagealreflux disease. Although H. pylori infection is associated with gastric cancer, no trials have shown thateradication of H. pylori purely to prevent gastric cancer is beneficial.

Ref: Ables AZ, Simon I, Melton ER: Update on Helicobacter pylori treatment. Am Fam Physician 2007;75(3):351-358.

Item 103

ANSWER: E

Fluoxetine is the only SSRI approved by the FDA for the treatment of depression in children 8–17 years ofage. Tricyclic antidepressants are ineffective in children and have limited effectiveness in adolescents, withsafety concerns in both groups. In children and adolescents, there is limited or no evidence evaluating theuse of lithium, monoamine oxidase inhibitors, St. John’s wort, or venlafaxine.

Ref: Bhatia SK, Bhatia SC: Childhood and adolescent depression. Am Fam Physician 2007;75(1):73-80.

Item 104

ANSWER: B

Polyarticular arthritis often presents with fever, knee and other joint effusions, and leukocytosis. A 24-hoururine collection is not routine, is difficult for the patient, and typically does not change therapy. Especiallyin cases where a joint effusion is accompanied by fever, diagnostic arthrocentesis should be performed tohelp guide therapy. Allopurinol should not be initiated during an acute gouty attack, but may be started aftera patient has recovered. Diuretics increase uric acid levels.

Page 36: 2008 It e Critique

35

Ref: Eggebeen AT: Gout: An update. Am Fam Physician 2007;76(6):801-808.

Item 105

ANSWER: B

After several unsuccessful attempts to remove an object deep in the ear canal of an uncooperative child, itis best to refer the patient to an otolaryngologist for removal under anesthesia. Additional attempts are veryunlikely to succeed, especially with the techniques listed. A loop curette cannot be safely placed behind aforeign body that is close to the tympanic membrane. A round, hard object cannot be grasped with forceps.Acetone can be used to dissolve Styrofoam foreign bodies, but it would not dissolve a plastic bead.

Ref: Heim SW, Maughan KL: Foreign bodies in the ear, nose, and throat. Am Fam Physician 2007;76(8):1185-1189.

Item 106

ANSWER: A

There are numerous options for the treatment of uterine fibroids. When pregnancy is desired, myomectomyoffers the best chance for a successful pregnancy when prior pregnancies have been marked byfibroid-related complications. Endometrial ablation eliminates fertility, and there is a lack of long-term dataon fertility after uterine artery embolization. Observation without treatment would not remove the risk forrecurrent complications during subsequent pregnancies.

Ref: Evans P, Brunsell S: Uterine fibroid tumors: Diagnosis and treatment. Am Fam Physician 2007;75(10):1503-1508.

Item 107

ANSWER: D

Multiple studies have demonstrated that bed rest is detrimental to recovery from low back pain. Patientsshould be encouraged to remain as active as possible. Exercises designed specifically for the treatment oflow back pain have not been shown to be helpful. Neither opioids nor trigger-point injections have shownsuperiority over placebo, NSAIDs, or acetaminophen in relieving acute back pain. There is no good evidenceto suggest that systemic corticosteroids are effective for low back pain with or without sciatica.

Ref: Kinkade S: Evaluation and treatment of acute low back pain. Am Fam Physician 2007;75(8):1181-1188.

Item 108

ANSWER: C

Although uncommon, pituitary disease can cause secondary hypothyroidism. The characteristic laboratory

4 4findings are a low serum free T and a low TSH. A free T level is needed to evaluate the proper dosage ofreplacement therapy in secondary hypothyroidism. The TSH level is not useful for determining the adequacyof thyroid replacement in this case, and the low level would prevent the physician from determining whetherthe dosage of levothyroxine is appropriate. In the initial evaluation of secondary hypothyroidism, a TRHstimulation test would be useful if TSH failed to rise in response to stimulation. It is not necessary in thiscase, since the diagnosis has already been made.

Page 37: 2008 It e Critique

36

Ref: Devdhar M, Ousman YH, Burman KD: Hypothyroidism. Endocrinol Metab Clin North Am 2007;36(3):595-615.

Item 109

ANSWER: A

This patient has dementia with Lewy bodies, which is the second most common histopathologic type ofdementia after Alzheimer’s disease. He demonstrates typical symptoms and signs of dementia with Lewybodies, including well-formed hallucinations, vivid dreams, fluctuating cognition, sleep disorder with periodsof daytime sleeping, frequent falls, deficits in visuospatial ability (abnormal clock drawing), and REM sleepdisorder (vivid dreams). In Alzheimer’s disease the predominant early symptom is memory impairment,without the other symptoms found in this patient. In dementia of Parkinson’s disease, extrapyramidalsymptoms such as tremor, bradykinesia, and rigidity precede the onset of memory impairment by more thana year. Patients with vascular dementia have risk factors and symptoms of stroke. Frontotemporal dementiapresents with behavioral changes, including disinhibition, or language problems such as various types ofaphasia.

Ref: Neef D, Walling AD: Dementia with Lewy bodies: An emerging disease. Am Fam Physician 2006;73(7):1223-1229, 1230.

Item 110

ANSWER: B

Several studies have demonstrated that hydration with sodium bicarbonate reduces the risk ofcontrast-induced nephropathy in those undergoing cardiac catheterization. Studies of interventions to preventrenal failure in patients at high risk have shown that mannitol plus hydration does not reduce acute renalfailure compared to hydration alone. Randomized, controlled trials have shown that fenoldopam does notdecrease the need for dialysis or improve survival. One systematic review found that low-osmolality contrastmedia reduced nephrotoxicity in persons with underlying renal failure requiring studies using contrast.

One systematic review and one subsequent randomized, controlled trial found that adding loop diuretics tofluids was not effective and may actually increase the possibility of acute renal failure compared to fluidsalone. A large randomized, controlled trial found no significant difference between natriuretic peptides andplacebo in preventing acute renal failure induced by contrast media.

Ref: Kellum J, Leblanc M, Venkataraman R: Acute renal failure. Am Fam Physician 2007;76(3):418-422. Ozcan EE, Guneri S,Akdeniz B, et al: Sodium bicarbonate, N-acetylcysteine, and saline for prevention of radiocontrast-induced nephropathy. Acomparison of 3 regimens for protecting contrast-induced nephropathy in patients undergoing coronary procedures. Asingle-center prospective controlled trial. Am Heart J 2007;154(3):539-544.

Item 111

ANSWER: B

Sensitivity is the ability of a test to identify patients who actually have the disease, or the true-positive rate.Independent of the sensitivity is the test’s specificity, which is the ability to correctly identify patients whodo not have the disease, or the true-negative rate. The greater the test’s specificity, the lower thefalse-positive rate; the greater the test’s sensitivity, the lower the false-negative rate.

Ref: Fauci AS, Braunwald E, Kasper DL, et al (eds): Harrison’s Principles of Internal Medicine, ed 17. McGraw-Hill, 2008, pp18-19.

Page 38: 2008 It e Critique

37

Item 112

ANSWER: A

This patient has moderate persistent asthma. The preferred and most effective treatment is daily inhaledcorticosteroids. A leukotriene inhibitor would be less effective. Oral prednisone daily is not recommendedbecause of the risk of inducing adrenal insufficiency. Short- and long-acting �-agonists are not recommendedas daily therapy because either can cause tachyphylaxis. They are considered rescue medications rather thanpreventive treatments.

Ref: Scow DT, Luttermoser GK, Dickerson KS: Leukotriene inhibitors in the treatment of allergy and asthma. Am Fam Physician2007;75(1):65-70. 2) National Asthma Education and Prevention Program: Expert Panel Report 3: Guidelines for theDiagnosis and Management of Asthma. National Heart, Lung, and Blood Institute, 2007. Available athttp://www.nhlbi.nih.gov/guidelines/asthma/asthgdln.pdf.

Item 113

ANSWER: B

Testosterone increases hematocrit and can cause polycythemia. In patients receiving testosteronesupplementation, hematocrit should be monitored every 6 months for the first 18 months, then annually.Testosterone should be discontinued if there is more than a 50% rise in hematocrit. Testosterone also causesan increase in lean body mass, and may increase bone density.

Ref: Margo K, Winn R: Testosterone treatments: Why, when, and how? Am Fam Physician 2006;73(9):1591-1598, 1603.

Item 114

ANSWER: A

The refusal to accept any medical intervention, including life-saving blood transfusions, has been wellestablished for adults who have the ability to definitively communicate their wishes. Also, parents have thepower to give or withhold consent to medical treatment on behalf of their children. However, Western courtshave deemed that parents cannot refuse emergency, life-saving treatment to children based on theseprinciples: (1) the child’s interests and those of the state outweigh parental rights to refuse medicaltreatment; (2) parental rights do not give parents life and death authority over their children; and (3) parentsdo not have an absolute right to refuse medical treatment for their children, if that refusal is regarded asunreasonable.

Ref: Woolley S: Jehovah’s Witnesses in the emergency department: What are their rights? Emerg Med J 2005;22(12):869-871.

Item 115

ANSWER: B

Amylase and lipase levels are used to help make the diagnosis of acute pancreatitis. The serum lipase levelis more specific and more sensitive than the amylase level. Amylase elevations can be seen with otherabdominal illnesses, such as inflammation of the small bowel. Alcoholics with recurrent pancreatitis mayhave normal serum amylase levels; in such cases, serum lipase would be a better test.

Page 39: 2008 It e Critique

38

There are several scoring systems for the severity of pancreatitis, including the CT severity index, theAPACHE II score, the Imrie Scoring System, and Ranson’s Criteria, but none of these use serum amylasein their calculation. The elevation of serum amylase does not correspond well with the severity of thepancreatitis.

Ref: Carroll JK, Herrick B, Gipson T, et al: Acute pancreatitis: Diagnosis, prognosis, and treatment. Am Fam Physician2007;75(10):1513-1520.

Item 116

ANSWER: D

Education, support, and medications are all valuable tools in assisting patients with a smoking habit.Varenicline, a clinically effective smoking-cessation product, has been associated with patient mood changesfollowing the initiation of therapy, including suicidal thoughts and aggressive and erratic behavior. The otherproblems listed have not been associated with varenicline use.

Ref: Public health advisory: Important information on Chantix. US Food and Drug Administration, 2008.

Item 117

ANSWER: C

Chronic paronychia is a common condition in workers whose hands are exposed to chemical irritants or arewet for long periods of time. This patient is an otherwise healthy hairdresser, with frequent exposure toirritants.

The patient should be advised to avoid exposure to harsh chemicals and water. In addition, the use of strongtopical corticosteroids over several weeks can greatly reduce the inflammation, allowing the nail folds toreturn to normal and helping the cuticles recover their natural barrier to infection.

Soaking in iodine solution would kill bacteria, but would also perpetuate the chronic irritation. Because thecondition is related to chemical and water irritation, a prolonged course of antibiotics should not be the firsttreatment step, and could have serious side effects. There is no need to explore less likely autoimmunecauses for nail changes at this time.

Ref: Rigopoulos D, Larios G, Gregoriou S, et al: Acute and chronic paronychia. Am Fam Physician 2008;77(3):339-346.

Item 118

ANSWER: A

This patient should have further testing for diabetes mellitus. Current recommendations for diagnosingdiabetes mellitus are based on either a fasting glucose level or a 2-hour 75-g oral glucose tolerance test. Acasual blood glucose level >200 mg/dL is also diagnostic of diabetes mellitus in patients with symptoms ofhyperglycemia. If unequivocal hyperglycemia is not present, the diagnosis must be confirmed by testing onanother day. Metformin can be used to treat diabetes mellitus in adolescents, but it is not recommended forprevention in this age group. A diagnosis of diabetes mellitus should be established prior to startingmetformin. Hospitalization is not required for this level of blood glucose unless ketoacidosis is present.Current recommendations for treating adolescents with type 2 diabetes mellitus include weight loss throughdietary modification and exercise.

Page 40: 2008 It e Critique

39

Ref: Peterson K, Silverstein J, Kaufman F, et al: Management of type 2 diabetes in youth: An update. Am Fam Physician2007;76(5):658-664. 2) American Diabetes Association: Diagnosis and classification of diabetes mellitus. Diabetes Care2008;31(Suppl 1):S55-S60.

Item 119

ANSWER: B

In terminally ill patients, the most common physical symptoms are pain, fatigue, and dyspnea. Opiates areuseful for controlling pain and relieving dyspnea as well. Even small doses of a weaker opiate can reducethe sensation of shortness of breath in cancer patients and in those with heart failure or chronic obstructivelung disease.

Addiction is rare in terminally ill patients who are being treated with opiates for pain and/or dyspnea. Amedication contract between physician and patient is not required by law and generally is not necessary inthis situation, unless diversion of the medication from the patient by the caregivers is suspected.Constipation due to decreased gastrointestinal motility is a very common, if not universal, side effect.Respiratory depression is a late, not early, sign of excessive opiate dosage. Another sign of opiate excess,pinpoint pupils, occurs before respiratory depression and is therefore a useful parameter for monitoring thesepatients.

Ref: Fauci AS, Braunwald E, Kasper DL, et al (eds): Harrison’s Principles of Internal Medicine, ed 17. McGraw-Hill, 2008, pp70-71.

Item 120

ANSWER: C

When giving bad news to a patient, it is important to assess how much information the patient wants to knowand tailor the discussion accordingly (SOR C). If the patient states that he does not want to know about theprognosis, the most appropriate response is to ask if he would like to talk again at another time. Thephysician may be regarded as rude, cruel, or uncaring if he persists in providing information that the patientis not ready to hear. In addition, the patient is more at risk for feelings of hopelessness, depression, oranxiety if he is not psychologically ready to hear a bad prognosis. Once the patient verbalizes a readinessto discuss the prognosis, specific information can be provided, focusing on both the positive and negativeaspects of the situation.

It is not appropriate to discuss the case with the patient’s wife, unless he specifically requests she be a proxyto receive the information. Referrals to either a support group or an oncologist should be deferred until theinformation has been discussed with the patient.

Ref: Ngo-Metzger Q, August KJ, Srinivasan M, et al: End-of-life care: Guidelines for patient-centered communication. Am FamPhysician 2008;77(2):167-174.

Item 121

ANSWER: D

This patient most likely is malingering, which is to purposefully feign physical symptoms for external gain.Factitious disorder involves adopting physical symptoms for unconscious internal gain, such as derivingcomfort from taking on the role of being sick. Somatization disorder is related to numerous unexplainedphysical symptoms that last for several years and typically begin before 30 years of age. Conversion disorderinvolves a single voluntary motor or sensory dysfunction suggestive of a neurologic condition, but not

Page 41: 2008 It e Critique

40

conforming to any known anatomic pathways or physiologic mechanisms.

Ref: American Psychiatric Association: Diagnostic and Statistical Manual of Mental Disorders, ed 4. American PsychiatricAssociation, 1994, p 683. 2) Oyama O, Paltoo C, Greengold J: Somatoform disorders. Am Fam Physician2007;76(9):1333-1338.

Item 122

ANSWER: B

The patient has Dupuytren’s disease, which is most common in men over 40 years of age. It is a progressivecondition that causes the fibrous fascia of the palmar surface to shorten and thicken. It initially can bemanaged with observation, but corticosteroid injection and surgery may be needed. The condition willregress in 10% of patients. There is a 3%–33% prevalence of Dupuytren’s contracture in patients withdiabetes mellitus; however, these patients tend to have a mild form of the disease with slow progression.

Ref: Trojian TH, Chu SM: Dupuytren’s disease: Diagnosis and treatment. Am Fam Physician 2007;76(1):86-89.

Item 123

ANSWER: D

Genetic disorders (e.g., Waardenburg syndrome, Usher’s syndrome, Alport syndrome, and Turner’ssyndrome) are responsible for more than 50% of hearing impairments in children. Intraventricularhemorrhage is a central cause of hearing loss, and is rare. Conductive abnormalities such as external canalanomalies and congenital cholesteatoma, and sensorineural causes other than genetic disorders (e.g.,infectious diseases) are important but less frequent.

Ref: Wrightson AS: Universal newborn hearing screening. Am Fam Physician 2007;75(9):1349-1352.

Item 124

ANSWER: E

Topical mupirocin is as effective as cephalexin or amoxicillin/clavulanate in the treatment of impetigo. Oralpenicillin V, oral erythromycin, and topical bacitracin are less effective than mupirocin. Topicaldisinfectants such as hydrogen peroxide are no more effective than placebo.

Ref: Cole C, Gazewood J: Diagnosis and treatment of impetigo. Am Fam Physician 2007;75(6):859-864.

Item 125

ANSWER: E

Alcohol withdrawal syndrome encompasses a wide range of symptoms involving primarily the centralnervous, cardiovascular, and gastrointestinal systems, and is mediated by the abrupt removal ofalcohol-enhanced GABA inhibition of excitatory glutamate receptors in the central nervous system. Itgenerally is divided into three stages, based on severity and timeline; seizures may occur during any of thesestages and may be the first sign of withdrawal.

Page 42: 2008 It e Critique

41

The ideal pharmacologic agent should provide not only safe sedation but also protection from seizures.Long-acting benzodiazepines such as chlordiazepoxide have been shown to be superior to the other choicesin numerous studies. Clonidine and atenolol have been found to be useful in symptom reduction but not inseizure prevention. Phenytoin would seem to offer protection from seizures, but studies have not consistentlyshown this to be the case. Phenobarbital, while effective, has a very narrow therapeutic window, making itsuse problematic.

Ref: Blondell RD: Ambulatory detoxification of patients with alcohol dependence. Am Fam Physician 2005;71(3):495-502.

Item 126

ANSWER: A

Rheumatoid arthritis is primarily a clinical diagnosis and no single laboratory test is considered definitivelydiagnostic. Anti-cyclic citrullinated peptide (anti-CCP) antibody is recommended by rheumatologists toimprove the specificity of testing for rheumatoid arthritis. Anti-CCP is more specific than rheumatoid factor,and may predict erosive disease more accurately.

Antinuclear antibody has limited usefulness for the diagnosis of rheumatoid arthritis. Sm antigen is usefulto help diagnose systemic lupus erythematosus. Nonspecific changes in complement levels are seen in manyrheumatologic disorders. The erythrocyte sedimentation rate is useful in monitoring disease activity and thecourse of rheumatoid arthritis, but is not specific.

Ref: Rindfleisch JA, Muller D: Diagnosis and management of rheumatoid arthritis. Am Fam Physician 2005;72(6):1037-1047.2) Nishimura K, Sugiyama D, Kogata Y, et al: Meta-analysis: Diagnostic accuracy of anti-cyclic citrullinated peptide antibodyand rheumatoid factor for rheumatoid arthritis. Ann Intern Med 2007;146(11):797-808.

Item 127

ANSWER: A

This child has a respiratory syncytial virus (RSV) infection. Supportive care is the mainstay of therapy. Ifthe child can take in fluids by mouth and tolerate room air, outpatient management with close physiciancontact as needed is reasonable, especially in the absence of significant underlying risk factors.

Routine use of corticosteroids is not recommended (SOR B). Although up to 60% of infants hospitalized forbronchiolitis receive corticosteroid therapy, studies have not provided sufficient evidence to support theiruse. Inhaled corticosteroids have not been shown to be beneficial, and the safety of high doses in infants is

2unclear. Supplemental oxygen should be administered if functional oxygen saturation (SpO ) persistentlyfalls below 90% and can be discontinued when an adequate level returns (SOR C).

Antiviral therapy for RSV bronchiolitis is controversial because of its marginal benefit, cumbersomedelivery, potential risk to caregivers, and high cost (SOR B). Studies of ribavirin in patients withbronchiolitis have produced inconsistent findings.

Palivizumab is a preventive measure, and is not used for treatment of the active disease. It may be consideredin select infants and children with prematurity, chronic lung disease of prematurity, or congenital heartdisease (SOR A). If used, it should be administered intramuscularly in five monthly doses of 15 mg/kg,usually beginning in November or December (SOR C).

Page 43: 2008 It e Critique

42

Ref: American Academy of Pediatrics Subcommittee on Diagnosis and Management of Bronchiolitis: Diagnosis and managementof bronchiolitis. Pediatrics 2006;118(4):1774-1793. 2) Krilov LR: Respiratory syncytial virus (RSV) infection. eMedicine2006. Available at http://www.emedicine.com/ped/topic2706.htm.

Item 128

ANSWER: B

Cautious reduction of systolic blood pressure by 10%–15% while monitoring neurologic status seems to bethe safest treatment goal in the setting of acute ischemic stroke when the systolic blood pressure is >220 mmHg or the diastolic blood pressure is 120–140 mm Hg. According to JNC-7, more aggressive blood pressurereduction may increase cerebrovascular complications.

Ref: The Seventh Report of the Joint National Committee on Prevention, Detection, Evaluation, and Treatment of High BloodPressure. National High Blood Pressure Education Program, 2004, NIH pub no 04-5230, p 38. 2) Adams HP Jr, del ZoppoG, Alberts MJ, et al: Guidelines for the early management of adults with ischemic stroke: A guideline from the AmericanHeart Association/American Stroke Association Stroke Council, Clinical Cardiology Council, Cardiovascular Radiology andIntervention Council, and the Atherosclerotic Peripheral Vascular Disease and Quality of Care Outcomes in ResearchInterdisciplinary Working Groups: The American Academy of Neurology affirms the value of this guideline as an educationaltool for neurologists. Stroke 2007;38(5):1655-1711.

Item 129

ANSWER: B

A small spontaneous pneumothorax involving less than 15%–20% of lung volume can be managed byadministering oxygen and observing the patient. The pneumothorax will usually resorb in about 10 days ifno ongoing air leak is present. Oxygen lowers the pressure gradient for nitrogen and favors transfer of gasfrom the pleural space to the capillaries. Decompression with anterior placement of an intravenous catheteris usually reserved for tension pneumothorax. Chest tube placement is used if observation is not successfulor for larger pneumothoraces. Strict bed rest is not indicated.

Ref: Marx JA (ed): Rosen’s Emergency Medicine: Concepts and Clinical Practice, ed 6. Mosby Elsevier, 2006, pp 1143-1147.

Item 130

ANSWER: E

Migraine is the most likely diagnosis in this scenario, because the patient is young and female; the headachesare unilateral, infrequent, and throbbing; the headaches are associated with nausea and vomiting; and sleepoffers relief. Symptoms of sinusitis usually include fever, facial pain, and a purulent nasal discharge. Thepain of cerebral tumor tends to occur daily and becomes more frequent and severe with time. Furthermore,the prevalence of brain tumor is far less than that of migraine. The pain of muscle tension headache isdescribed as a pressure or band-like tightening, often in a circumferential or cap distribution. This headachealso has a pattern of daily persistence, often continuing day and night for long periods of time. Clusterheadache is more common in males, and presents as a constant, agonizing orbital pain, usually beginningwithin 2 or 3 hours after falling asleep.

Ref: Ropper AH, Brown RH: Adams and Victor’s Principles of Neurology, ed 8. McGraw-Hill, 2005, pp 147-165.

Page 44: 2008 It e Critique

43

Item 131

ANSWER: A

There is little evidence that femoral anteversion causes long-term functional problems. Studies have shownthat shoe wedges, torque heels, and twister cable splints are not effective. Surgery should be reserved forchildren 8–10 years of age who still have cosmetically unacceptable, dysfunctional gaits. Majorcomplications of surgery occur in approximately 15% of cases, and can include residual in-toeing, out-toeing,avascular necrosis of the femoral head, osteomyelitis, fracture, valgus deformity, and loss of position. Thus,observation alone is appropriate treatment for a 5-year-old with uncomplicated anteversion.

Ref: Sass P, Hassan G: Lower extremity abnormalities in children. Am Fam Physician 2003;68(3):461-468. 2) Kliegman RM,Behrman RE, Jenson HB, et al (eds): Nelson Textbook of Pediatrics, ed 18. Saunders, 2007, pp 2784-2785, 2786.

Item 132

ANSWER: B

Many antibiotics can induce pseudomembranous colitis. Although oral vancomycin was once the initial drugof choice for C. difficile, oral metronidazole is now the first-line agent because of cost considerations andbecause of concerns about the development of vancomycin-resistant organisms. If the patient has refractorysymptoms despite treatment with oral metronidazole, then oral vancomycin would be appropriate.

Ref: Goldman L, Ausiello D (eds): Cecil Medicine, ed 23. Saunders, 2008, p 2203.

Item 133

ANSWER: E

The pain from infiltration of local anesthetics can be decreased by using a warm solution, using smallneedles, performing the infiltration slowly, and adding sodium bicarbonate to the mixture. It also helps toinject the agent through the edges of the wound (assuming the wound is not contaminated) and to pretreatthe wound with topical anesthetics.

Ref: Singer AJ, Hollander JE, Quinn JV: Evaluation and management of traumatic lacerations. N Engl J Med1997;337(16):1142-1148. 2) Marx JA (ed): Rosen’s Emergency Medicine: Concepts and Clinical Practice, ed 6. MosbyElsevier, 2006, pp 2930-2933.

Item 134

ANSWER: B

Continued oral feeding in diarrhea aids in recovery, and an age-appropriate diet should be given.Breastfeeding or regular formula should be continued.

Foods with complex carbohydrates (e.g., rice, wheat, potatoes, bread, and cereals), lean meats, yogurt, fruits,and vegetables are well tolerated. Foods high in simple sugars (e.g., juices, carbonated sodas) should beavoided because the osmotic load can worsen the diarrhea. Fatty foods should be avoided as well. TheBRAT diet has not been shown to be effective.

Page 45: 2008 It e Critique

44

Ref: Managing acute gastroenteritis among children. MMWR 2003;52(RR16):1-16. 2) Kliegman RM, Behrman RE, Jenson HB,et al (eds): Nelson Textbook of Pediatrics, ed 18. Saunders, 2007, pp 1613-1615.

Item 135

ANSWER: D

Because the growth pattern of the fetus varies throughout pregnancy, the accuracy of measurements and theirusefulness in determining gestational age and growth vary with each trimester. Crown-rump length is thedistance from the top of the head to the bottom of the fetal spine. It is most accurate as a measure ofgestational age at 7–14 weeks. After that, other measurements are more reliable. In the second trimester,biparietal diameter and femur length are used. During the third trimester, biparietal diameter, abdominalcircumference, and femur length are best for estimating gestational age.

Ref: Cunningham FG, Leveno KJ, Bloom SL, et al: Williams Obstetrics, ed 22. McGraw-Hill, 2005, p 392.

Item 136

ANSWER: B

Thrombosis of the upper extremity accounts for about 10% of all venous thromboembolism (VTE) cases.However, axillosubclavian vein thrombosis (ASVT) is becoming more frequent with the increased use ofindwelling subclavian vein catheters. Spontaneous ASVT (not catheter related) is seen most commonly inyoung, healthy individuals. The most common associated etiologic factor is the presence of a compressiveanomaly in the thoracic outlet. These anomalies are often bilateral, and the other upper extremity at similarrisk for thrombosis.

While a hypercoagulable state also may contribute to the thrombosis, it is much less common. Budd-Chiarisyndrome refers to thrombosis in the intrahepatic, suprahepatic, or hepatic veins. It is not commonlyassociated with spontaneous upper-extremity thrombosis.

Ref: Joffe HV, Goldhaber SZ: Upper-extremity deep vein thrombosis. Circulation 2002;106(14):1874-1880. 2) Marx JA (ed):Rosen’s Emergency Medicine: Concepts and Clinical Practice, ed 6. Mosby Elsevier, 2006, pp 1358-1360.

Item 137

ANSWER: E

For geriatric patients in long-term care facilities, the predictable glucose control of glargine is the bestapproach to consider initially. The American Diabetes Association does not recommend a strict diet for fraildiabetic patients in nursing homes. Exenatide is not recommended for the frail elderly because of concernsabout weight loss and nausea. Heart failure precludes the use of pioglitazone, and renal failure precludesthe use of metformin.

Ref: Mazza AD, Morley JE: Update of diabetes in the elderly and the application of current therapeutics. J Am Med Dir Assoc2007;8(8):489-492.

Page 46: 2008 It e Critique

45

Item 138

ANSWER: B

The U.S. Preventive Services Task Force has released a statement summarizing recommendations forscreening for abdominal aortic aneurysm (AAA). The guideline recommends one-time screening withultrasonography for AAA in men 65–75 years of age who have ever smoked. No recommendation was madefor or against screening women. Men with a strong family history of AAA should be counseled about therisks and benefits of screening as they approach 65 years of age.

Ref: Upchurch GR, Schaub TA: Abdominal aortic aneurysm. Am Fam Physician 2006;73(7):1198-1204.

Item 139

ANSWER: E

This patient is already receiving maximal medical therapy. The 2002 joint guidelines of the AmericanCollege of Cardiology, the American Heart Association (AHA), and the North American Society of Pacingand Electrophysiology endorse the use of cardiac resynchronization therapy (CRT) in patients with medicallyrefractory, symptomatic, New York Heart Association (NYHA) class III or IV disease with a QRS intervalof at least 130 msec, a left ventricular end-diastolic diameter of at least 55 mm, and a left ventricular ejectionfraction (LVEF) �30%. These guidelines were refined by an April 2005 AHA Science Advisory, whichstated that “optimal candidates for CRT have a dilated cardiomyopathy on an ischemic or nonischemic basis,an LVEF �0.35, a QRS complex �120 msec, and sinus rhythm, and are NYHA functional class III or IVdespite maximal medical therapy for heart failure.”

Ref: Jarcho JA: Biventricular pacing. N Engl J Med 2006;355(3):288-294.

Item 140

ANSWER: E

Pancreatitis is most closely associated with gallstones and excessive alcohol use. Gastroesophageal refluxdisease, pyelonephritis, drug abuse (other than alcohol), and angiotensin receptor blocker use are not riskfactors for the development of pancreatitis.

Ref: Whitcomb DC: Acute pancreatitis. N Engl J Med 2006;354(20):2142-2150.

Item 141

ANSWER: E

Bacterial vaginosis is the most common cause of acute vaginitis, accounting for up to 50% of cases in somepopulations. It is usually caused by a shift in normal vaginal flora. Mixed vaginal flora is considerably morecommon as a cause of vaginal discharge than C. albicans and T. vaginalis.

Ref: Eckert LO: Acute vulvovaginitis. N Engl J Med 2006;355(12):1244-1252.

Page 47: 2008 It e Critique

46

Item 142

ANSWER: D

The symptom of an increased or abnormal sensation of one’s heartbeat is referred to as “palpitations.” Thiscondition is common to primary care, but is often benign. Commonly, these sensations have their basis inanxiety or panic. However, about 50% of those who complain of palpitations will be found to have adiagnosable cardiac condition. It is recommended to start the evaluation for cardiac causes with an EKG,which will assess the baseline rhythm and screen for signs of chamber enlargement, previous myocardialinfarction, conduction disturbances, and a prolonged QT interval.

Ref: Abbott AV: Diagnostic approach to palpitations. Am Fam Physician 2005;71(4):743-750.

Item 143

ANSWER: A

Pain involving the big toe is a common problem. The first metatarsophalangeal (MTP) joint has twosesamoid bones, and injuries to these bones account for 12% of big-toe injuries. Overuse, a sharp blow, andsudden dorsiflexion are the most common mechanisms of injury.

Gout commonly involves the first MTP joint, but the onset is sudden, with warmth, redness, and swelling,and pain on movement of the joint is common. Morton’s neuroma commonly causes numbness involvingthe digital nerve in the area, and usually is caused by the nerve being pinched between metatarsal heads inthe center of the foot. Cellulitis of the foot is common, and can result from inoculation through a subtlecrack in the skin. However, there would be redness and swelling, and the process is usually moregeneralized.

Sesamoiditis is often hard to differentiate from a true sesamoid fracture. Radiographs should be obtained,but at times they are nondiagnostic. Treatment, fortunately, is similar, unless the fracture is open or widelydisplaced. Limiting weight bearing and flexion to control discomfort is the first step. More complextreatments may be needed if the problem does not resolve in 4–6 weeks.

Ref: Brukner P, Khan K: Clinical Sports Medicine, ed 3. McGraw Hill, 2006, pp 667-668, 861.

Item 144

ANSWER: B

This patient has classic signs of hypothyroidism. Of the drugs listed, only lithium is associated with thedevelopment of hypothyroidism. In patients taking lithium, it is recommended that in addition to regular

4serum lithium levels, thyroid function tests including total free T , and TSH be obtained yearly.

Ref: Griswold KS, Pessar LF: Management of bipolar disorder. Am Fam Physician 2000;62(6):1343-1353, 1357-1358. 2)Kronenberg HM, Melmed S, Polonsky KS, et al(eds): Williams Textbook of Endocrinology, ed 11. Saunders Elsevier, 2008,pp 392-393.

Page 48: 2008 It e Critique

47

Item 145

ANSWER: D

Whole cow’s milk does not supply infants with enough vitamin E, iron, and essential fatty acids, andoverburdens their system with too much protein, sodium, and potassium. Skim and low-fat milk lead to thesame problems as whole milk, and also fail to provide adequate calories for growth. For these reasons cow’smilk is not recommended for children under 12 months of age. Human breast milk or iron-fortified formula,with introduction of certain solid foods and juices after 4–6 months of age if desired, is appropriate for thefirst year of life.

Ref: Allen RE, Myers AL: Nutrition in toddlers. Am Fam Physician 2006;74(9):1527-1532.

Item 146

ANSWER: C

According to the Beers criteria, a list of drugs that should generally be avoided in older adults, meperidineshould not be used in the elderly because its metabolite can accumulate and cause seizures. The othermedications are not listed in the Beers criteria and are not contraindicated in the elderly.

Ref: Pham CB, Dickman RL: Minimizing adverse drug events in older patients. Am Fam Physician 2007;76(12):1837-1844.

Item 147

ANSWER: D

In patients with renal failure, the risk for death and serious cardiovascular events is increased with higherhemoglobin levels (�13.5 g/dL), and it is therefore recommended that levels be maintained at 10–12 g/dL.Studies have also demonstrated less mortality and morbidity when the dosage of epoetin alfa is set to achievea target hemoglobin of <12 g/dL.

Ref: Singh AK, Szczech L, Tang KL, et al: Correction of anemia with epoetin alfa in chronic kidney disease. N Engl J Med2006;355(20):2085-2098.

Item 148

ANSWER: B

In 2004 the American Academy of Pediatrics published updated clinical practice guidelines on themanagement of hyperbilirubinemia in the newborn infant at 35 or more weeks gestation. These guidelinesfocus on frequent clinical assessment of jaundice, and treatment based on the total serum bilirubin level, theinfant’s age in hours, and risk factors. Phototherapy should not be started based solely on the total serumbilirubin level. The guidelines encourage breastfeeding 8–12 times daily in the first few days of life toprevent dehydration. There is no evidence to support supplementation with water or dextrose in water in anondehydrated breastfeeding infant. This infant is not dehydrated and is getting an adequate number offeedings, and there is no reason to discontinue breastfeeding at this time.

Ref: American Academy of Pediatrics Subcommittee on Hyperbilirubinemia: Management of hyperbilirubinemia in the newborninfant 35 or more weeks of gestation. Pediatrics 2004;114(1):297-316.

Page 49: 2008 It e Critique

48

Item 149

ANSWER: A

Oral iron is absorbed better with an acidic gastric environment, which can be accomplished with theconcomitant administration of vitamin C. Agents that raise gastric pH, such as antacids, proton pump

2inhibitors, and H blockers, should be avoided if possible. Oral iron absorption is improved if the iron istaken on an empty stomach, but this may not be well tolerated because gastric irritation is a frequent sideeffect. Constipation also is common with oral iron therapy. Iron therapy should be continued for severalmonths after the hemoglobin reaches a normal level, in order to fully replenish iron stores. Sustained-releaseoral iron products provide a decreased amount of iron for absorption.

Ref: Killip S, Bennett JM, Chambers MD: Iron deficiency anemia. Am Fam Physician 2007;75(5):671-678.

Item 150

ANSWER: A

This is a classic description of rickettsial illness: a history of outdoor activity, the sudden onset of fever,chills, and rash on the palms of the hands and the soles of the feet. Penicillin, cephalexin, and azithromycindo not cover rickettsia. Reassurance would be inappropriate because this condition can be life threateningand should always be treated.

Ref: Huntzinger A: Practice guidelines: Guidelines for the diagnosis and treatment of tick-borne rickettsial diseases. Am FamPhysician 2007;76(1):137.

Item 151

ANSWER: C

The diagnosis of schizophrenia requires two or more of the following characteristic symptoms (each presentfor a significant portion of time during a 1-month period): delusions, hallucinations, disorganized speech,grossly disorganized or catatonic behavior, and negative symptoms (i.e., affective flattening, alogia, oravolition). In addition, one or more major areas of functioning, such as work, interpersonal relationships,or self-care, should be markedly below the level seen prior to the onset of symptoms. Schizoaffective andmood disorders, substance abuse, medical illness or medication-induced disorders, and pervasivedevelopmental disorders should be ruled out.

Brief psychotic disorder is characterized by the presence of delusions, hallucinations, disorganized speech,or grossly disorganized or catatonic behavior lasting at least 1 day but less than 1 month. Delirium maypresent with psychotic symptoms but is the direct physiologic consequence of a general medical conditionand usually has a much shorter course. Mood disorders with psychotic features can be ruled out if no majordepressive, manic, or mixed episodes have occurred concurrently with the active-phase symptoms, or if theduration of mood disturbance is brief compared to the overall duration of active and residual symptoms.Delusional disorder does not cause bizarre delusions and also lacks other characteristic symptoms ofschizophrenia such as hallucinations, disorganized speech or behavior, or prominent negative symptoms.

Ref: American Psychiatric Association: Diagnostic and Statistical Manual of Mental Disorders, ed 4. American PsychiatricAssociation, 1994, pp 274-286. 2) Schultz SH, North SW, Shields CG: Schizophrenia: A review. Am Fam Physician2007;75(12):1821-1829.

Page 50: 2008 It e Critique

49

Item 152

ANSWER: A

This patient meets the criteria for metabolic syndrome. In addition to lifestyle changes, pharmacologictreatment for his hyperlipidemia should be considered. The initial goal of this therapy should be to reachhis LDL-cholesterol goal, usually using a statin. After achievement of this goal, non-HDL cholesterol is thesecondary target for therapy. Non-HDL cholesterol is calculated by subtracting HDL cholesterol from totalcholesterol. The non-HDL cholesterol goal is 30 mg/dL higher than the LDL-cholesterol goal.

Ref: Oh RC, Lanier JB: Management of hypertriglyceridemia. Am Fam Physician 2007;75(9):1365-1371, 1372.

Item 153

ANSWER: D

No medication available in the United States has been shown to effectively treat cough or cold symptomsin children younger than 2 years of age. However, many agents are commonly prescribed despite reports ofnumerous minor, and some serious, adverse effects. For this reason, the American Academy of Pediatricsand the American Academy of Family Physicians recommend using only nasal saline, bulb suction,humidified air, and good hydration in children younger than 2 years of age.

Ref: Simasek M, Blandino DA: Treatment of the common cold. Am Fam Physician 2007;75(4):515-520.

Item 154

ANSWER: B

The times published for Current Procedural Terminology codes are typical for each level of office visit, butthere is no requirement associated with them unless counseling or coordination of care accounts for morethan 50% of the face-to-face time of the encounter. In such cases physicians are able to code on the basisof time.

Ref: Hughes C: Coding and documentation. Fam Pract Manage 2006;13(10):27.

Item 155

ANSWER: C

Development of pleural plaques is the most common pathologic pulmonary response to asbestos inhalation.Over time, collagen is deposited in the pleura and may calcify. Most plaques are asymptomatic, and thereis no evidence that plaques transform into malignant lesions. Plaques occur in approximately 50% of personswith heavy and prolonged exposure to asbestos and, therefore, are a marker of asbestos exposure. Thisshould alert the physician to follow the patient for development of more serious asbestos-related diseases(e.g., lung cancer and mesothelioma). Although the other substances listed are associated with pulmonarydiseases (coal dust and silicon dust with pneumoconiosis, and vinyl chloride and radon with lung cancer),none is associated with pleural plaques as found in this patient.

Ref: O’Reilly KM, McLaughlin AM, Beckett WS, et al: Asbestos-related lung disease. Am Fam Physician 2007;75(5):683-688.2) Collins LG, Haines C, Perkel R, et al: Lung cancer: Diagnosis and management. Am Fam Physician 2007;75(1):56-63.

Page 51: 2008 It e Critique

50

Item 156

ANSWER: B

This patient has pleurisy. Patients presenting with pleuritic chest pain may have life-threatening disorders,and pulmonary embolism, acute myocardial infarction, and pneumothorax should be excluded. While5%–20% of patients with pulmonary embolism present with pleuritic chest pain, this patient has no risks forpulmonary embolism and the normal D-dimer level obviates the need for further evaluation. Moderate- tohigh-risk patients may need a helical CT scan or other diagnostic testing.

An EKG and chest radiograph are recommended in the evaluation of acute/subacute pleuritic chest pain. Thechest radiograph will exclude pneumothorax, pleural effusion, or pneumonia. An echocardiogram would notbe indicated if the cardiac examination and EKG are normal. An antinuclear antibody level could beconsidered in recurrent pleurisy or if other symptoms or signs of lupus were present, but it would not beindicated in this patient.

Most cases of acute pleurisy are viral and should be treated with NSAIDs unless the workup indicatesanother problem.

Ref: Kass SM, Williams PM, Reamy BV: Pleurisy. Am Fam Physician 2007;75(9):1357-1364.

Item 157

ANSWER: D

The chest radiograph of a child with meconium aspiration syndrome will show patchy atelectasis orconsolidation. If the child has a normal chest film and respiratory distress, a noncardiopulmonary sourceshould be considered (i.e., a neurologic or metabolic etiology). The chest film of a child with transienttachypnea of the newborn will show a “wet silhouette” around the heart, diffuse parenchymal infiltrates, orintralobar fluid accumulation. Homogeneous opaque infiltrates with air bronchograms on a chest radiographare seen with hyaline membrane disease.

Ref: Hermansen CL, Lorah KN: Respiratory distress in the newborn. Am Fam Physician 2007;76(7):987-994.

Item 158

ANSWER: D

Factors associated with a higher risk of heat-related deaths include being confined to bed, not leaving homedaily, and being unable to care for oneself. Living alone during a heat wave is associated with an increasedrisk of death, but this increase is not statistically significant. Among medical conditions, the highest risk isassociated with preexisting psychiatric illnesses, followed by cardiovascular disease, use of psychotropicmedications, and pulmonary disease.

A lower risk of heat-related deaths has been noted in those who have working air conditioning, visitair-conditioned sites, or participate in social activities. Those who take extra showers or baths and who usefans have a lower risk, but this difference is not statistically significant.

Ref: Bouchama A, Dehbi M, Mohamed G, et al: Prognostic factors in heat wave–related deaths: A meta-analysis. Arch Intern Med2007;167(20):2170-2176.

Page 52: 2008 It e Critique

51

Item 159

ANSWER: D

This adolescent has findings of Marfan syndrome. It is associated with arachnodactly, arm span greater thanheight, a high arched palate, kyphosis, lenticular dislocation, mitral valve prolapse, myopia, and pectusexcavatum. Cardiac examination may reveal an aortic insufficiency murmur, or a murmur associated withmitral valve prolapse. Cardiovascular defects are progressive, and aortic root dilation occurs in 80%–100%of affected individuals. Aortic regurgitation becomes more common with increasing age.

Ref: Kliegman RM, Behrman RE, Jenson HB, et al (eds): Nelson Textbook of Pediatrics, ed 18. Saunders, 2007, pp 2890-2893.2) Giese EA, O’Connor FG, Depenbrock PJ, et al: The athletic preparticipation evaluation: Cardiovascular assessment. AmFam Physician 2007;75(7):1008-1014.

Item 160

ANSWER: B

Intrahepatic cholestasis of pregnancy classically presents as severe pruritus in the third trimester.Characteristic findings include the absence of primary skin lesions and elevation of serum levels of total bileacids. Jaundice and elevated bilirubin levels may or may not be present. The GGT usually is normal ormodestly elevated, which can help differentiate this condition from other cholestatic liver diseases. Theprothrombin time usually is normal, but if elevated it may reflect a vitamin K deficiency from malabsorption.

Ref: Riely CA: Liver disease in the pregnant patient. Am J Gastroenterol 1999;94(7):1728-1732. 2) Tunzi M, Gray GR: Commonskin conditions during pregnancy. Am Fam Physician 2007;75(2):211-218. 3) Gabbe SG, Niebyl JR, Simpson JL (eds):Obstetrics: Normal and Problem Pregnancies, ed 5. Churchill Livingstone, 2007, p 112.

Item 161

ANSWER: B

This patient has unilateral serous otitis and is unlikely to have delayed language from decreased hearing onone side. The patient should be observed for now. Hearing loss of longer than 3 months may indicate a needfor tympanostomy tubes. Surgical treatment has been shown to be helpful, but should be reserved forpatients with chronic problems. Audiometry is not needed to make a decision about surgery at this point,and the mother’s judgment is likely correct about his current hearing loss, so a hearing test most likely wouldnot add any useful information. Numerous studies have debunked all medical treatments for serous otitis,including antihistamine and decongestant therapy, and corticosteroids by any route.

Ref: Ramakrishnan K, Sparks RA, Berryhill WE: Diagnosis and treatment of otitis media. Am Fam Physician2007;76(11):1650-1658.

Item 162

ANSWER: B

Ultrasonography of the right upper quadrant is recommended as the initial imaging study for right upperquadrant pain (SOR C). KUB films can detect kidney stones but may miss gallstones. CT also may missgallstones, and is more invasive than ultrasonography. Abdominal CT with intravenous contrast is thepreferred test for right lower quadrant pain, and abdominal CT with intravenous and oral contrast is preferredfor left lower quadrant pain. MRI is preferred for detecting tumors, and is inappropriate as the initial imagingstudy for right upper quadrant pain.

Page 53: 2008 It e Critique

52

Ref: Cartwright SL, Knudson MP: Evaluation of acute abdominal pain in adults. Am Fam Physician 2008;77(7):971-978.

Item 163

ANSWER: C

The primary goals of atrial fibrillation treatment are rate control and prevention of thromboembolism.Guidelines recommend rate control with atenolol, metoprolol, diltiazem, or verapamil (SOR A).

Digoxin does not control the heart rate with stress. Quinidine is proarrhythmic and does not control the heartrate. Nifedipine does not control the heart rate, and heparin does not provide adequate anticoagulation orcontrol the heart rate.

Ref: Snow V, Weiss KB, LeFevre M, et al: Management of newly detected atrial fibrillation: A clinical practice guideline fromthe American Academy of Family Physicians and the American College of Physicians. Ann Intern Med2003;139(12):1009-1017. 2) Huntzinger A: Joint guideline released for atrial fibrillation. Am Fam Physician2007;75(10):1565-1568.

Item 164

ANSWER: E

Leukoplakia is a white keratotic lesion seen on mucous membranes. Irritation from various mechanical andchemical stimuli, including alcohol, favors development of the lesion. Leukoplakia can occur in any areaof the mouth and usually exhibits benign hyperkeratosis on biopsy. On long-term follow-up, 2%–6% of theselesions will have undergone malignant transformation into squamous cell carcinoma. Oral nystatin wouldnot be appropriate treatment, as this lesion is not typical of oral candidiasis. Candidal lesions are usuallymultiple and spread quickly when left untreated. A fluorescent antinuclear antibody test is also not indicated,as the oral lesions of lupus erythematosus are typically irregular, erosive, and necrotic. An idiosyncraticreaction to propranolol is unlikely in this patient.

Ref: Gonsalves WC, Chi AC, Neville BW: Common oral lesions: Part II. Masses and neoplasia. Am Fam Physician2007;75(4):509-512. 2) Goldman L, Ausiello D (eds): Cecil Medicine, ed 23. Saunders, 2008, pp 1451-1452.

Item 165

ANSWER: A

The clinical triad of Horner’s syndrome—ipsilateral ptosis, miosis, and decreased facial sweating—suggestsdecreased sympathetic innervation due to involvement of the stellate ganglion, a complication of Pancoast’ssuperior sulcus tumors of the lung. Radiographs or MRI of the pulmonary apices and paracervical area isindicated.

Horner’s syndrome may accompany intracranial pathology, such as the lateral medullary syndrome(Wallenberg’s syndrome), but is associated with multiple other neurologic symptoms, so MRI of the brainis not indicated at this point.

Page 54: 2008 It e Critique

53

The acetylcholine receptor antibody level is a test for myasthenia gravis, which can also present with ptosis,but not with full-blown Horner’s syndrome. Diabetes mellitus and thyroid disease do not commonly presentwith Horner’s syndrome.

Ref: Goldman L, Ausiello D (eds): Cecil Medicine, ed 23. Saunders, 2008, p 2796.

Item 166

ANSWER: A

Metformin increases insulin sensitivity much more than sulfonylureas or insulin. This means lower insulinlevels achieve the same level of glycemic control, and may be one reason that weight changes are less likelyto be seen in diabetic patients on metformin. Acarbose is an �-glucosidase inhibitor that delays glucoseabsorption.

Ref: Fauci AS, Braunwald E, Kasper DL, et al (eds): Harrison’s Principles of Internal Medicine, ed 17. McGraw-Hill, 2008, pp2299-2302.

Item 167

ANSWER: D

Despite this individual’s busy schedule, he has a potentially life-threatening problem that needs properdiagnosis and treatment. Though an excisional biopsy takes longer, it is the procedure of choice whenmelanoma is suspected. After removal and diagnosis, prompt referral is essential for further evaluation andtherapy. A shave biopsy should never be done for suspected melanoma, as this is likely to transect the lesionand destroy evidence concerning its depth, thus making it difficult to assess the prognosis. A punch biopsyshould be used only with discretion when the lesion is too large for complete excision, or if substantialdisfigurement would occur. Since this may not actually retrieve cancerous tissue from an unsampled areaof a large lesion that might be malignant, it would be safest to refer such patients. Neither cryotherapy norelectrocautery should be used for a suspected melanoma.

Ref: Habif TP, Campbell JL Jr, Chapman MS, et al: Skin Disease: Diagnosis and Treatment, ed 2. Elsevier, 2005, pp 472-479.

Item 168

ANSWER: E

Volvulus may present in one of three ways: as a sudden onset of bilious vomiting and abdominal pain in aneonate; as a history of “feeding problems” with bilious vomiting that appears to be a bowel obstruction; orless commonly, as failure to thrive with severe feeding intolerance. The classic finding on abdominal plainfilms is the “double bubble” sign, which shows a paucity of gas (airless abdomen) with two air bubbles, onein the stomach and one in the duodenum. However, the plain film can be entirely normal. The uppergastrointestinal contrast study is considered the gold standard for diagnosing volvulus.

Infantile colic usually begins during the second week of life and typically occurs in the evening. It ischaracterized by screaming episodes and a distended or tight abdomen. Its etiology has yet to be determined.There are no abnormalities on physical examination and ancillary studies, and symptoms usually resolvespontaneously around 12 weeks of age.

Page 55: 2008 It e Critique

54

Necrotizing enterocolitis is typically seen in the distressed neonate in the intensive-care nursery, but it mayoccasionally be seen in the healthy neonate within the first 2 weeks of life. The child will appear ill, withsymptoms including irritability, poor feeding, a distended abdomen, and bloody stools. Abdominal plainfilms will show pneumatosis intestinalis, caused by gas in the intestinal wall, which is diagnostic of thecondition.

Hypertrophic pyloric stenosis is a narrowing of the pyloric canal caused by hypertrophy of the musculature.It usually presents during the third to fifth weeks of life. Projectile vomiting after feeding, weight loss, anddehydration are common. The vomitus is always nonbilious, because the obstruction is proximal to theduodenum. If a small olive-size mass cannot be felt in the right upper or middle quadrant, ultrasonographywill confirm the diagnosis.

Intussusception is seen most frequently between the ages of 3 months and 5 years, with 60% of casesoccurring in the first year and a peak incidence at 6–11 months of age. The disorder occurs predominantlyin males. The classic triad of intermittent colicky abdominal pain, vomiting, and bloody, mucous stools isencountered in only 20%–40% of cases. At least two of these findings will be present in approximately 60%of patients. The abdomen may be distended and tender, and there may be an elongated mass in the rightupper or lower quadrants. Rectal examination may reveal either occult blood or frankly bloody,foul-smelling stool, classically described as “currant jelly.” An air enema using fluoroscopic guidance isuseful for both diagnosis and treatment.

Ref: McCollough M, Sharieff GQ: Abdominal pain in children. Pediatr Clin North Am 2006;53(1):107-137.

Item 169

ANSWER: A

Commotio cordis usually results from impact with a projectile in sports. Children and adolescents may haveincreased risk due to a compliant chest wall. Ventricular fibrillation is thought to result from the impact.Softer “safety” baseballs are one consideration in primary prevention.

Older competitors are at less risk. Large blunt objects or body-to-body contact also carries less risk.

Ref: Maron BJ, Gohman TE, Kyle SB, et al: Clinical profile and spectrum of commotio cordis. JAMA 2002;287(9):1142-1146.2) Zangwill SD, Strasburger JF: Commotio cordis. Pediatr Clin North Am 2004;51(5):1347-1354. 3) Maron BJ, Estes NAIII, Link MS: Task force 11: Commotio cordis. J Am Coll Cardiol 2005;45(8):1371-1373.

Item 170

ANSWER: B

Aggressive control of blood pressure to <135/85 mm Hg in hypertensive patients and to <130/80 mm Hg indiabetic patients is recommended. Lowering blood pressure may reduce stroke rates by 40%–52% andcardiovascular morbidity by 18%–20%.

Ref: Abrams J, Thadani U: Therapy of stable angina pectoris: The uncomplicated patient. Circulation 2005;112(3):e257-e258.2) The Seventh Report of the Joint National Committee on Prevention, Detection, Evaluation, and Treatment of High BloodPressure. National High Blood Pressure Education Program, 2004, NIH Publication No. 04-5230, pp 25, 36-37.

Page 56: 2008 It e Critique

55

Item 171

ANSWER: A

There is not a linear correlation between bone mineral density and fracture risk. Bone architecture may bechanged by bisphosphonate therapy, which may result in a decreased fracture risk. This patient has not hada fracture and is on adequate medical therapy that should be continued.

Ref: Boonen S: Bisphosphonate efficacy and clinical trials for postmenopausal osteoporosis: Similarities and differences. Bone2007;3(2):S26-S31.

Item 172

ANSWER: B

A Cochrane review including randomized, controlled trials (RCTs) compared standard access to emergencycontraception (EC) with advance provision. The review found eight trials, five of which were conducted inthe U.S. Two of the RCTs were sufficiently powered to show a difference in pregnancy rates. No studyshowed that giving advance EC reduced pregnancy rates on a population level. However, women who wereprovided with advance EC took the pills an average of approximately 15 hours sooner than women withoutadvance access.

Five studies that reported on contraception use did not show a difference in type or frequency of regularcontraception use among women who were provided advance EC. Women randomized to the advance ECgroups were 2.5 times more likely to use EC once, and 4 times more likely to use it 2 or more times,compared to those without advance access.

Three studies reported rates of sexually transmitted infection and none found differences between theadvance and standard access EC groups. Six studies reported rates of unprotected sexual intercourse andfound no difference.

The Cochrane review concludes that advance access to EC appears to be safe, but does not reduce pregnancyon a population level. However, advance provision might be beneficial because it increases the speed andfrequency of EC use.

Ref: Emergency contraception. American College of Obstetricians and Gynecologists, Practice Bulletin no 69, 2005. 2) AmericanAcademy of Pediatrics Committee on Adolescence: Emergency contraception. Pediatrics 2005;116(4):1026-1035. 3) CripkeC: Advance provision for emergency oral contraception. Am Fam Physician 2007;76(5):654.

Item 173

ANSWER: C

Red cell distribution width (RDW) is particularly useful in distinguishing anemic disorders, especially irondeficiency anemia (high RDW, normal to low mean corpuscular volume) and uncomplicated heterozygousthalassemia (normal RDW, low mean corpuscular volume).

Ref: Rudolph CD, Rudolph AM (eds): Rudolph’s Pediatrics, ed 21. McGraw-Hill, 2003, pp 1523, 1528.

Page 57: 2008 It e Critique

56

Item 174

ANSWER: A

The live, attenuated influenza vaccine (LAIV) is an option for vaccinating healthy, nonpregnant individualsage 5–49 years. The vaccine is administered intranasally. It is not indicated in patients with underlyingmedical conditions, such as chronic pulmonary or cardiovascular disease, or in patients with a history ofGuillain-Barré syndrome, pregnant patients, or children and adolescents who receive long-term aspirin orsalicylate therapy. Patients with a history of hypersensitivity to eggs also should not receive this vaccine.

Ref: Prevention and control of influenza: Recommendations of the Advisory Committee on Immunization Practices (ACIP)—2007.MMWR 2007;56(RR-6):1-54.

Item 175

ANSWER: E

The adaptation that permits survival in chronic hyponatremia also makes the brain vulnerable to injury fromoverzealous therapy. When hyponatremia is corrected too rapidly, outpacing the brain’s ability to recapturelost organic osmolytes, osmotic demyelination can result. Osmotic demyelination syndrome can usually beavoided by limiting correction of chronic hyponatremia to <10–12 mmol/L in 24 hours and to <18 mmol/Lin 48 hours.

Ref: Verbalis JG, Goldsmith SR, Greenburg A, et al: Hyponatremia treatment guidelines 2007: Expert panel recommendations.Am J Med 2007;120(11 Suppl 1):S1-S21.

Item 176

ANSWER: A

In general, the strongest evidence for treatment, screening, or prevention strategies is found in systematicreviews, meta-analyses, randomized controlled trials (RCTs) with consistent findings, or a single high-qualityRCT. Second-tier levels of evidence would be poorer quality RCTs with inconsistent findings, cohortstudies, or case-control studies. The lowest quality of evidence would come from such sources as expertopinion, consensus guidelines, or usual practice recommendations.

Ref: Ebell MH, Siwek J, Weiss BD, et al: Strength of recommendation taxonomy (SORT): A patient-centered approach to gradingevidence in the medical literature. Am Fam Physician 2004;69(3):548-556.

Item 177

ANSWER: B

This patient has a significant pneumonia that requires the initiation of empiric antibiotics. It is important toremember that because this patient was recently in the hospital, the usual coverage for community-acquiredpneumonia is not adequate. Health care–associated pneumonia is more likely to involve severe pathogenssuch as Pseudomonas aeruginosa, Klebsiella pneumoniae, and Acinetobacter species. Methicillin-resistantStaphylococcus aureus also is a consideration, depending on local prevalence. Of the antibiotic regimenslisted, ceftazidime and gentamicin is the only choice that covers these organisms.

Page 58: 2008 It e Critique

57

Ref: American Thoracic Society, Infectious Diseases Society of America: Guidelines for the management of adults withhospital-acquired, ventilator-associated, and healthcare-associated pneumonia. Am J Respir Crit Care Med2005;171(4):388-416.

Item 178

ANSWER: E

For unknown reasons, Streptococcus bovis bacteremia or endocarditis is associated with a high incidenceof occult colorectal malignancies. It may also occur with upper gastrointestinal cancers. Radiography orendoscopy is indicated.

Ref: Gold JS, Bayar S, Salem RR: Association of Streptococcus bovis bacteremia with colonic neoplasia and extracolonicmalignancy. Arch Surg 2004;139(7):760-765. 2) Fauci AS, Braunwald E, Kasper DL, et al (eds): Harrison's Principles ofInternal Medicine, ed 17. McGraw-Hill, 2008, p 575.

Item 179

ANSWER: E

The American Urological Association (AUA) defines clinically significant microscopic hematuria as �3RBCs/hpf. Microscopic hematuria is frequently an incidental finding, but may be associated with urologicmalignancy in up to 10% of adults.

The upper urinary tract should be evaluated in this patient. There are no clear evidence-based imagingguidelines for upper tract evaluation; therefore, intravenous urography, ultrasonography, or CT can beconsidered. Ultrasonography is the least expensive and safest choice because it does not expose the patientto intravenous radiographic contrast media.

Urine cytology and cystoscopy are used routinely to evaluate the lower urinary tract. The AUA recommendsthat patients with microscopic hematuria have radiographic assessment of the upper urinary tract, followedby urine cytology studies. The AUA also recommends that all patients older than 40 and those who areyounger but have risk factors for bladder cancer undergo cystoscopy to complete the evaluation. Cystoscopyis the only reliable method of detecting transitional cell carcinoma of the bladder and urethra.

Ref: McDonald MM, Swagerty D, Wetzel L: Assessment of microscopic hematuria in adults. Am Fam Physician2006;73(10):1748-1754, 1759.

Item 180

ANSWER: D

The use of herbal and nutritional supplements has become commonplace in the United States. Unfortunately,there is insufficient research for most herbal remedies, in terms of both efficacy and safety. However, thereis a significant body of evidence from randomized, controlled trials and various meta-analyses showingbenefit from the use of kava in the short-term treatment of anxiety disorders (up to 24 weeks), includinggeneralized anxiety disorder (SOR A). The other remedies listed have only single studies or anecdotalevidence attesting to benefit for patients with anxiety. At best, information about them is limited, and thereare often conflicting results.

Page 59: 2008 It e Critique

58

Safety concerns about kava have been addressed by recent randomized, controlled trials demonstrating thatkava has a safety profile similar to those of FDA-approved treatments for anxiety disorders. Care should betaken with any concurrent use of kava and medications metabolized by the liver, and patients should bediscouraged from using alcohol while taking kava.

Physicians should be aware of all remedies their patients are taking, even if they are not prescribed. Inaddition, it is important to be aware of remedies that have evidence supporting their use.

Ref: Saeed SA, Bloch RM, Antonacci DJ: Herbal and dietary supplements for treatment of anxiety disorders. Am Fam Physician2007;76(4):549-556.

Item 181

ANSWER: E

Low-dose topical, oral, nasal, and intra-articular corticosteroids are not immunosuppressive and do notcontraindicate administration of any vaccine. Influenza vaccine and pneumococcal vaccine can be giventogether. Neither is a live vaccine.

Ref: Zimmerman RK, Middleton DB: Vaccines for persons at high risk, 2007. J Fam Pract 2007;56(2 Suppl Vaccines):S38-S46,C4-C5.

Item 182

ANSWER: B

This patient is suffering from severe COPD (stage III) and has a history of frequent exacerbations. Theaddition of a corticosteroid inhaler for patients with severe disease has been found to significantly decreasethe number of exacerbations, but has no effect on overall mortality. Side effects of oral candidiasis and easybruising of the skin are increased.

Continuous oxygen has been shown to improve overall mortality and endurance in patients with an oxygensaturation of 88% or less, but has not been shown to improve quality of life in those with mild hypoxemiaor if used only at night.

Oral prednisone has been shown to be effective when used to treat acute exacerbations, but when used ona chronic basis it is no more effective than corticosteroid inhalers. Chronic oral prednisone is also associatedwith significant side effects, and therefore is not generally recommended.

Oral theophylline has not been shown to be of benefit in either preventing exacerbations or improving qualityof life, and has significant side effects of gastrointestinal toxicity, seizures, and arrhythmias. It should bereserved for carefully selected patients only.

Ref: Grimes GC, Manning JL, Patel P, et al: Medications for COPD: A review of effectiveness. Am Fam Physician2007;76(8):1141-1148. 2) Global Initiative for Chronic Obstructive Lung Disease, Global Strategy for the Diagnosis,Management, and Prevention of Chronic Obstructive Pulmonary Disease, 2007.

Page 60: 2008 It e Critique

59

Item 183

ANSWER: E

Many medications require dosage adjustments in patients with chronic kidney disease. Medications areadjusted based on the estimated glomerular filtration rate (GFR) or creatinine clearance. Most medicationadjustments require a reduction in the dose, lengthening of the dosing interval, or both. Loading doses ofmedications usually do not need to be adjusted.

Medication adjustments are divided into three groups, based on whether the GFR is >50 mL/min/1.73m ,2

10–50 mL/min/1.73m , or <10 mL/min/1.73m . The production and excretion of creatinine decreases in2 2

older patients, so a normal serum creatinine level does not always correlate with normal kidney function.Serum drug levels typically are not required for adjusting medications in patients with chronic kidneydisease.

Ref: Munar MY, Singh H: Drug dosing adjustments in patients with chronic kidney disease. Am Fam Physician2007;75(10):1487-1496.

Item 184

ANSWER: E

This child has atopic dermatitis (eczema). It is manifested by a pruritic rash on the face and/or extensorsurfaces of the arms and/or legs, especially in children. There often is a family history of atopy or allergies.In addition to the regular use of emollients, the mainstay of maintenance therapy, topical corticosteroids havebeen shown to be the best first-line treatment for flare-ups of atopic dermatitis. Topical calcineurin inhibitorsshould be second-line treatment for flare-ups, but are not recommended for use in children under 2 years ofage. Antibiotics should be reserved for the treatment of acutely infected lesions. There is no evidence tosupport the use of topical anesthetics or analgesics in the treatment of this disorder.

Ref: Buys LM: Treatment options for atopic dermatitis. Am Fam Physician 2007;75(4):523-528.

Item 185

ANSWER: A

(Note: The publication of the ONTARGET study after this examination went to press indicates that none ofthe answers listed is an appropriate choice, and all answers were therefore counted as correct. See Lancet2008;372(9638):547-553.)

There are several strategies for slowing the progression of chronic kidney disease to end-stage renal disease,which requires either a renal transplant or dialysis. ACE inhibitors have been shown to be renoprotectiveby preventing the progression of diabetic nephropathy. There also is data showing that angiotensin receptorblockers probably are as effective as ACE inhibitors. Combining the two can further retard the progress ofchronic kidney disease, but with the caveat that the two medications can increase the risk of hyperkalemiain a patient with impaired renal function, and the combination confers a higher mortality risk than eitheragent alone.

In addition to tight glucose control, cessation of smoking, weight control, and treatment of hyperlipidemia,patients should strive to control hypertension. If proteinuria is less than 1 g/day, then the blood pressure goalshould be 130/80 mm Hg; however, if proteinuria is more than 1 g/day, some authorities recommend a goalof 125/75 mm Hg. While good control of blood pressure is ideal, calcium channel blockers (especially thedihydropyridine class) should be used only if necessary, because they have been associated with a more rapiddecline of glomerular filtration rate.

Page 61: 2008 It e Critique

60

Recent studies have shown that patients on a low-protein diet have a slower rate of deterioration in renalstatus. The recommended restriction is 0.6–0.8 g/kg/day.

Anemia of chronic disease is a manifestation of chronic renal disorder. Although erythropoietin therapy maynot prevent progression, it does improve survival and quality of life.

Ref: Zandi-Nejad K, Brenner BM: Strategies to retard the progression of chronic kidney disease. Med Clin North Am2005;89(3):489-509.

Item 186

ANSWER: D

In a patient with sepsis, vasopressors are indicated when fluid resuscitation does not restore organ perfusionand blood pressure. Norepinephrine and dopamine currently are the preferred pressor agents; however,norepinephrine appears to be more effective and has a lower mortality rate. Phenylephrine, epinephrine, orvasopressin should not be used as first-line therapy. Vasopressin is employed after high-dose conventionalvasopressors have failed. The use of low-dose dopamine is no longer recommended based on a clinical trialshowing no benefit in critically ill patients at risk for renal failure. If an agent is needed to increase cardiacoutput, dobutamine is the agent of choice.

Ref: Dellinger RP, Carlet JM, Masur H, et al: Surviving Sepsis Campaign guidelines for management of severe sepsis and septicshock. Crit Care Med 2004;32(3):858-873. 2) Khan ZU, Salzman GA: Management of sepsis: The surviving sepsisguidelines for early therapy. Hosp Physician 2007;43(6):21-30.

Item 187

ANSWER: A

Strabismus is an ocular misalignment that can be diagnosed on a cover/uncover test when the corneal lightreflex is deviated from its normal position slightly nasal to mid-pupil. The misaligned eye then moves tofixate on a held object when the opposite eye is covered. The eye drifts back to its original position whenthe opposite eye is uncovered.

Amblyopia is a visual impairment from abnormal visual development—most often as a result of strabismus.Cataract is a less frequent cause of amblyopia. Esotropia is a type of strabismus with an inward or nasaldeviation of the eye that would be evidenced by a corneal light reflex lateral to its normal position. (Theoutward eye deviation seen in this patient is exotropia.) Heterophoria, or latent strabismus, does not causeeye deviation when both eyes are uncovered.

Ref: Doshi NR, Rodriguez ML: Amblyopia. Am Fam Physician 2007;75(3):361-367.

Item 188

ANSWER: A

Pneumococcal and meningococcal vaccines are currently recommended for patients with asplenia.Haemophilus influenzae type b (Hib) vaccine can be considered as well. A Td booster or Tdap also shouldbe considered. Emergency splenectomy for trauma is an indication for vaccination, although splenicremnants may persist.

Page 62: 2008 It e Critique

61

Ref: Zimmerman RK, Middleton DB: Vaccines for persons at high risk, 2007. J Fam Pract 2007;56(2 Suppl Vaccines):S38-S46,C4-C5.

Item 189

ANSWER: B

The risk associated with abnormal results on cytologic testing vary with the age of the patient. Adolescentshave a high prevalence of human papillomavirus (HPV) infection, but a very low risk for invasive cervicalcancer. The vast majority of HPV infections will clear within 2 years after the initial infection, and have littleclinical significance. Follow-up annual cytologic testing is the only recommended management foradolescents with ASC-US. Only those whose smears are classified as high-grade squamous intraepitheliallesions (HSIL) or higher on repeat cytology should be referred for colposcopy at the 12-month follow-up.At the 24-month follow-up, those with smears classified as ASC-US or higher should be referred forcolposcopy (AII). HPV DNA testing is inappropriate for adolescents, and LEEP is not recommended at anyage for women with ASC-US.

Ref: Wright TC Jr, Massad LS, Dunton CJ, et al: 2006 consensus guidelines for the management of women with abnormal cervicalcancer screening tests. Am J Obstet Gynecol 2007;197(4):346-355.

Item 190

ANSWER: D

For previously healthy patients with community-acquired pneumonia and no risk factors for drug resistance,a macrolide such as azithromycin is the preferred treatment (SOR A). Doxycycline is also acceptable (SORC).

Patients who have been treated with antibiotics within the previous 3 months should be treated with arespiratory fluoroquinolone (moxifloxacin, gemifloxacin, or levofloxacin) (SOR A). A �-lactam plus amacrolide is also an alternative (SOR A). The antibiotic chosen should be from a different class than the oneused for the previous infection. These alternative treatments are also recommended for those withcomorbidities such as chronic heart, lung, liver, or renal disease; diabetes mellitus; alcoholism; malignancies;asplenia; immunosuppressing conditions or use of immunosuppressing drugs; or other risk factors fordrug-resistant Streptococcus pneumoniae infection (SOR A).

Ref: Mandell LA, Wunderink RG, Anzueto A, et al: Infectious Diseases Society of America/American Thoracic Society consensusguidelines on the management of community-acquired pneumonia in adults. Clin Infect Dis 2007;44(Suppl 2):S27-S72.

Item 191

ANSWER: E

Hydroxychloroquine, originally developed as an antimalarial drug, is a well-known disease-modifying agentthat can slow the progression of rheumatoid arthritis. Aspirin, indomethacin, and ibuprofen areanti-inflammatory agents. They relieve pain and improve mobility, but do not alter the progression of thedisease. Capsaicin, a topical substance-P depleter, can relieve pain symptoms.

Ref: Klippel JH, Stone JH, Crofford LJ, et al (eds): Primer on the Rheumatic Diseases, ed 13. Springer, 2008, pp 134-140.

Page 63: 2008 It e Critique

62

Item 192

ANSWER: D

Colic is a frustrating condition for parents and doctors alike. The parents would like an explanation andrelief, and physicians would like to offer these things. At this time, however, in spite of numerous studiesand theories, the cause of colic remains unknown.

Ref: Miller-Loncar C, Bigsby R, High P, et al: Infant colic and feeding difficulties. Arch Dis Child 2004;89(10):908-912.

Item 193

ANSWER: A

The criteria for diagnosing diabetes mellitus include any one of the following: symptoms of diabetes(polyuria, polydipsia, weight loss) plus a casual glucose level �200 mg/dL; a fasting plasma glucose level�126 mg/dL; or a 2-hour postprandial glucose level �200 mg/dL after a 75-gram glucose load. In the absenceof unequivocal hyperglycemia the test must be repeated on a different day.

The criteria for impaired glucose homeostasis include either a fasting glucose level of 100–125 mg/dL(impaired fasting glucose) or a 2-hour glucose level of 140–199 mg/dL on an oral glucose tolerance test.Normal values are now considered <100 mg/dL for fasting glucose and <140 mg/dL for the 2-hour glucoselevel on an oral glucose tolerance test.

Ref: American Diabetes Association: Diagnosis and classification of diabetes mellitus. Diabetes Care 2008;31(Suppl 1):S55-S60.

Item 194

ANSWER: A

Pleural effusions may be exudates or transudates. The distinction is important for an accurate diagnosis andto help determine what further evaluations may be necessary. Light’s criteria use ratios of fluid/serum valuesfor protein and LDH. Pleural fluid/serum ratios greater than 0.6 for LDH and 0.5 for protein are indicativeof exudates. In the scenario presented, both ratios are approximately 0.3; therefore, the fluid is a transudate.

The list of causes for transudates is much shorter than for exudates. The vast majority of transudates are dueto heart failure, with cirrhosis being the next most common cause. Once there is reasonable certainty thatthe fluid is a transudate, additional studies usually are not necessary. The other conditions listed result inexudative pleural effusions.

Ref: Porcel JM, Light RT: Diagnostic approach to pleural effusion in adults. Am Fam Physician 2006;73(7):1211-1220.

Item 195

ANSWER: A

Enoxaparin is eliminated mostly by the kidneys. When it is used in patients with severe renal impairmentthe dosage must be significantly reduced. For some indications the dose normally given every 12 hours isgiven only every 24 hours.

Page 64: 2008 It e Critique

63

Although some �-blockers require a dosage adjustment, metoprolol and carvedilol are metabolized by theliver and do not require dosage adjustment in patients with renal failure. Clopidogrel is currentlyrecommended at the standard dosage for patients with renal failure and acute coronary syndrome.Thrombolytics are given at the standard dosage in renal failure, although hemorrhagic complications areincreased.

Ref: O’Hanlon R, Reddan DN: Treatment of acute coronary syndromes in patients who have chronic kidney disease. Med ClinNorth Am 2005;89(3):563-585. 2) Field JM, Hazinski MF, Gilmore D (eds): Handbook of Emergency Cardiovascular Carefor Healthcare Providers. American Heart Association, 2006, pp 45-63.

Item 196

ANSWER: D

Nightmares occur in the second half of the night, when rapid eye movement (REM) sleep is most prominent.Parasomnias are disorders of arousal from non-REM (NREM) sleep. These are more common in childrenthan adults because children spend more time in deep NREM sleep. Such disorders usually occur within 1–2hours after sleep onset, and coincide with the transition from the first period of slow-wave sleep. Thesedisorders include sleepwalking, confusional arousal, and sleep terrors.

Ref: Moore M, Allison D, Rosen CL: A review of pediatric nonrespiratory sleep disorders. Chest 2006;130(4):1252-1262.

Item 197

ANSWER: E

Risk factors for osteoarthritis of the hip include obesity, high bone mass, old age, participation inweight-bearing sports, and hypothyroidism.

Ref: Lane NE: Osteoarthritis of the hip. N Engl J Med 2007;357(14):1413-1421.

Item 198

ANSWER: B

Obstructive sleep apnea-hypopnea syndrome is defined as the presence of at least five obstructive events perhour with associated daytime sleepiness. It is present in 2%–4% of the population. The prevalence in menis almost three times that seen in premenopausal women and twice that of postmenopausal women. Otherfactors associated with an increased prevalence are obesity, older age, and systemic hypertension.

Ref: Basner RC: Continuous positive airway pressure for obstructive sleep apnea. N Engl J Med 2007;356(17):1751-1758.

Item 199

ANSWER: B

Physicians should avoid reacting to laboratory values without considering the clinical scenario. This patientpresented with mild dehydration and normal laboratory values. Although he is improving clinically, hislaboratory values show multiple unexpected results. The most noticeable is the severely elevated glucose,because he has no history of diabetes mellitus or use of medications that could cause this effect. Similarly,the elevated potassium and decreased sodium suggest profound electrolyte abnormalities. Most likely, thelaboratory technician drew blood from the patient’s indwelling port without discarding the first several

Page 65: 2008 It e Critique

64

milliliters. Thus, the blood was contaminated with intravenous fluids, resulting in the erroneous results. Arepeat blood test from a peripheral vein should give more accurate results.

Ref: Daniels R: Delmar’s Guide to Laboratory and Diagnostic Tests. Delmar, 2002, pp 788-790.

Item 200

ANSWER: D

2Quetiapine is an atypical antipsychotic that has no clinically significant effect on the dopamine D receptor,which is responsible for the parkinsonian side effects of antipsychotic medications. Because of this, it isconsidered the antipsychotic of choice in patients with dementia associated with Parkinson’s disease,although its use has not been studied extensively in this clinical situation. The other atypical antipsychotics

2listed, olanzapine and risperidone, have some D receptor effect. Thioridazine and haloperidol are typicalantipsychotics and have more side effects, including parkinsonian side effects; they are not recommendedin this clinical situation.

Ref: Rayner AV, O’Brien JG, Schoenbachler B: Behavior disorders of dementia: Recognition and treatment. Am Fam Physician2006;73(4):647-652, 653-654. 2 ) Reilly TH, Kirk MA: Atypical antipsychotics and newer antidepressants. Emerg Med ClinNorth Am 2007;25(2):477-497.

Item 201

ANSWER: E

Risk factors for proximal neoplasia include high-grade dysplasia, three or more adenomas, adenomas withvillous features, and an adenoma �1 cm in size. For patients with one or more of these findings, follow-upcolonoscopy in 3 years is recommended. The clinical benefit of follow-up surveillance colonoscopy inpatients with one or two small adenomas has never been demonstrated. Distal hyperplastic polyps are notmarkers for proximal or advanced neoplasia. Patients with this finding on colonoscopy should be consideredto have a normal colonoscopy and the interval until the next colonoscopy should be 10 years.

Ref: Atkin WS, Morson BC, Cuzick J: Long-term risk of colorectal cancer after excision of rectosigmoid adenomas. N Engl J Med1992;326(10):658-662. 2) Lin OS, Schembre DB, McCormick SE, et al: Risk of proximal colorectal neoplasia amongasymptomatic patients with distal hyperplastic polyps. Am J Med 2005;118(10):1113-1119. 3) Winawer SJ, Zauber AG,Fletcher RH, et al: Guidelines for colonoscopy—surveillance after polypectomy. CA Cancer J Clin 2006;56(3):142-147. 4)Levin TR: Reducing unnecessary surveillance colonoscopies: A mandate for endoscopists. Gastrointest Endosc2006;63(1):104-106.

Item 202

ANSWER: D

A patient with a recurrent kidney stone and an elevated serum calcium level most likely hashyperparathyroidism, and a parathyroid hormone (PTH) level would be appropriate. Elevated PTH is causedby a single parathyroid adenoma in approximately 80% of cases. The resultant hypercalcemia is oftendiscovered in asymptomatic persons having laboratory work for other reasons. An elevated PTH byimmunoassay confirms the diagnosis. In the past, tests based on renal responses to elevated PTH were usedto make the diagnosis. These included blood phosphate, chloride, and magnesium, as well as urinary ornephrogenous cyclic adenosine monophosphate. These tests are not specific for this problem, however, andare therefore not cost-effective. Serum calcitonin levels have no practical clinical use.

Page 66: 2008 It e Critique

65

Ref: Fauci AS, Braunwald E, Kasper DL, et al (eds): Harrison’s Principles of Internal Medicine, ed 17. McGraw-Hill, 2008, pp2380-2383.

Item 203

ANSWER: A

This patient likely has gout. Aspiration should be attempted to get a specific diagnosis. The initial treatmentfor gout is NSAIDs, colchicine, or cortisone injections (SOR B). Allopurinol should be avoided until theepisode of gout is controlled, because it may precipitate worsening. In addition to medication, recommendedmanagement includes addressing risk factors such as obesity, diuretic use, high-purine diet, and alcoholintake (SOR B).

Ref: Eggebeen AT: Gout: An update. Am Fam Physician 2007;76(6):801-808.

Item 204

ANSWER: A

The rotavirus vaccine is an oral vaccine recommended for infants in a 3-dose schedule at ages 2, 4, and 6months. The first dose should be given between 6 and 12 weeks of age, with additional doses given at 4- to10-week intervals. The vaccine cannot be initiated after 12 weeks of age and should not be administeredafter 32 weeks of age.

The first rotavirus vaccine was taken off the market in 1999 after an increased risk of intussusception wasnoted in infants when the first dose of the vaccine was given after 12 weeks of age. Because of this, the twovaccines licensed in 2006 carry the recommendation that the vaccine not be initiated in infants over 12 weeksof age. There is no data regarding safety and efficacy after this age.

Ref: American Academy of Pediatrics Committee on Infectious Diseases: Recommended immunization schedules for children andadolescents—United States, 2007. Pediatrics 2007;119(1):207-208.

Item 205

ANSWER: D

Somatization disorders should be considered in patients who have a history of various complaints over aseveral-year period that involve multiple organ systems. There is no test to confirm this diagnosis. It is oftenintertwined in other psychiatric problems, including anxiety disorder, personality disorder, and depression.Treatment includes testing to make sure that there is nothing physically wrong, while building a trustingrelationship with the patient. Once this is accomplished, it is reasonable to discuss the disorder with thepatient. Cognitive therapy has been shown to be of value, as well as regularly scheduled office visits formonitoring and support. Medicines for coexisting psychiatric problems also are of benefit. In addition,referral for psychiatric consultation may be worthwhile.

Food allergies can cause a variety of symptoms, but usually not to the extent seen with this patient, andtesting for this might confuse the issue. Lorazepam may help the symptoms if there is a coexisting anxietydisorder, but it will not address the underlying problem. Laryngeal esophageal reflux can cause hoarsenessand will respond to proton pump inhibitors, but given the repetitive nature of her symptoms and the previousnegative workups, it is not consistent with the whole picture.

Page 67: 2008 It e Critique

66

Ref: Oyama O, Paltoo C, Greengold J: Somatoform disorders. Am Fam Physician 2007;76(9):1333-1338.

Item 206

ANSWER: D

Lung cancer is the leading cause of cancer-related deaths in the United States. In 2006, lung cancer causedmore deaths than colorectal, breast, and prostate cancers combined.

Ref: Collins LG, Haines C, Perkel R, et al: Lung cancer: Diagnosis and management. Am Fam Physician 2007;75(1):56-63.

Item 207

ANSWER: D

Hospital management of acute exacerbations of asthma should include inhaled short-acting bronchodilatorsin all patients. Systemic corticosteroids are recommended for all patients admitted to the hospital. Theefficacy of oral prednisone has been shown to be equivalent to that of intravenous methylprednisolone (SORA). Oxygen should also be considered in most patients. Antibiotics are not recommended in the treatmentof asthma exacerbations unless there is a comorbid infection. Inhaled ipratropium bromide is recommendedfor treatment in the emergency department, but not in the hospital (SOR A). Chest physical therapy andmethylxanthines are not recommended in the treatment of acute asthma exacerbations.

Ref: National Asthma Education and Prevention Program: Expert panel report 3: Guidelines for the diagnosis and managementof asthma. National Heart, Lung, and Blood Institute, 2007, pp 373-417. Available at http://www.nhlbi.nih.gov/guidelines/asthma/asthgdln.pdf

Item 208

ANSWER: B

For patients with a first episode of unprovoked deep venous thrombosis, evidence supports treatment witha vitamin K antagonist for at least 3 months (SOR A). The American College of Chest Physiciansrecommends that patients be evaluated at that point for the potential risks and benefits of long-term therapy(SOR C).

Ref: Ebell M: Adult Medicine Update. AAFP Home Study Self-Assessment monograph series, 2006, no 325, pp 13-19. 2)Schraibman IG, Milne AA, Royle EM: Home versus in-patient treatment for deep vein thrombosis. Cochrane Database SystRev 2007(2):CD003076. 3) Hirsh J, Guyatt G, Albers GW, et al: Executive summary: American College of Chest PhysiciansEvidence-Based Clinical Practice Guidelines (8th Edition). Chest 2008;133(6 Suppl):71S-109S.

Item 209

ANSWER: B

Prevention traditionally has been divided into three categories: primary, secondary, and tertiary. Primaryprevention targets individuals who may be at risk to develop a medical condition and intervenes to preventthe onset of that condition (e.g., childhood vaccination programs, water fluoridation, antismoking programs,and education about safe sex). Secondary prevention targets individuals who have developed anasymptomatic disease and institutes treatment to prevent complications (e.g., routine Papanicolaou smears,and screening for hypertension, diabetes, or hyperlipidemia). Tertiary prevention targets individuals witha known disease, with the goal of limiting or preventing future complications (e.g., screening diabetics formicroalbuminuria, rigorous treatment of diabetes mellitus, and post–myocardial infarction prophylaxis with

Page 68: 2008 It e Critique

67

�-blockers and aspirin).

Ref: Roadmaps for clinical practice: A primer on population-based medicine. American Medical Association, 2002, pp 28-33.

Item 210

ANSWER: A

The ophthalmopathy of Graves’ disease may initially flare and worsen when treated with radioactive iodine.Antithyroid drugs, including carbimazole, propylthiouracil, and methimazole, are not associated with thisproblem. The addition of thyroid hormone to these drugs at suppressive doses has not shown any clearbenefit over titration of the antithyroid drug, and relapse rates are similar. Thyroid surgery in the controlledpatient has not been significantly associated with this problem.

Ref: Larsen PR, Kronenberg HM, Malmed S, et al (eds): Williams Textbook of Endocrinology, ed 10. Saunders, 2003, pp 393-401.2) Nygaard B: Clinical evidence concise: Hyperthyroidism. Am Fam Physician 2007;76(7):1014-1016.

Item 211

ANSWER: D

Common neurologic disorders in young women include multiple sclerosis, Guillain-Barré syndrome, andmyasthenia gravis. Myasthenia gravis is part of the differential diagnosis for sudden neurologic weakness,and Guillain-Barré syndrome must also be considered in this patient. Multiple sclerosis would not result inrespiratory compromise.

Myasthenia gravis is an autoimmune neuromuscular disease characterized by varying degrees of skeletalmuscle weakness. Symptoms, which vary in type and severity, may include ptosis of one or both eyelids;blurred vision; diplopia; unstable gait; weakness in the arms, hands, fingers, legs, and neck; difficultyswallowing; shortness of breath; and impaired speech (dysarthria). In most cases, the first noticeablesymptom is weakness of the eye muscles. Muscles that control respiration and neck and limb movementsmay also be affected. Symptoms typically worsen through the day or as the muscles are repetitively used,and improve with rest.

Fibromyalgia does not produce objective neurologic findings, and Sheehan’s syndrome would not cause alocalized neurologic deficit. In addition, the TSH level would be low or zero, and the MRI of the brain wouldbe abnormal. An MRI of the brain would also be abnormal if stroke symptoms had been present for 2 weeks.

The patient is unlikely to have unilateral symptoms with polymyositis, and creatine kinase would be elevated.

Ref: Dhand UK: Clinical approach to the weak patient in the intensive care unit. Respir Care 2006;51(9):1024-1040. 2)Conti-Fine BM, Milani M, Kaminski HJ: Myasthenia gravis: Past, present, and future. J Clin Invest 2006;116(11):2843-2854.

Page 69: 2008 It e Critique

68

Item 212

ANSWER: E

Keratoacanthoma grows rapidly and may heal within 6 months to a year. Squamous cell carcinoma mayappear grossly and histologically similar to keratoacanthoma but does not heal spontaneously. The otherlesions do not resemble keratoacanthoma.

Ref: Habif TP, Campbell JL Jr, Chapman MS, et al: Skin Disease: Diagnosis and Treatment, ed 2. Elsevier, 2005, pp 404-407.

Item 213

ANSWER: A

The ankle-brachial index (ABI) is an inexpensive, sensitive screening tool and is the most appropriate firsttest for peripheral vascular occlusive disease (PVOD) in this patient. The ABI is the ratio of systolic bloodpressure measured in the ankle to systolic pressure using the standard brachial measurement. A ratio of0.9–1.2 is considered normal. Severe disease is defined as a ratio <0.50.

More invasive and expensive testing using Doppler ultrasonography, arteriography, or magnetic resonanceangiography may be useful if the ABI suggests an abnormality. Venous ultrasonography would not detectPVOD, but it could rule out deep venous thrombosis, which is another common etiology for calf pain.

Ref: Sontheimer DL: Peripheral vascular disease: Diagnosis and treatment. Am Fam Physician 2006;73(11):1971-1976.

Item 214

ANSWER: C

Although essential hypertension is most common in adolescents and adults, it is rarely found in children lessthan 10 years old and should be a diagnosis of exclusion. The most common cause of hypertension is renalparenchymal disease, and a urinalysis, urine culture, and renal ultrasonography should be ordered for allchildren presenting with hypertension. Other secondary causes, such as pheochromocytoma,hyperthyroidism, and excessive caffeine use, are less common, and further testing and/or investigation shouldbe ordered as clinically indicated.

Ref: Luma GB, Spiotta RT: Hypertension in children and adolescents. Am Fam Physician 2006;73(9):1558-1566.

Item 215

ANSWER: B

The syndrome of inappropriate antidiuresis (SIAD, formerly SIADH) is related to a variety of pulmonary andcentral nervous system disorders in which hyponatremia and hypo-osmolality are paradoxically associatedwith an inappropriately concentrated urine. Most, but not all, cases are associated with increased levels ofthe antidiuretic hormone arginine vasopressin (AVP). For a diagnosis of SIAD to be made, the patient mustbe euvolemic and not on diuretics (within 24–48 hours), and the urine osmolality must be high in conjunctionwith both low serum sodium and low osmolality. The BUN is normal or low and the fractional excretion ofsodium is >1%.

Page 70: 2008 It e Critique

69

Fluid restriction (<800 cc/24 hours) over several days will correct the hyponatremia/hypo-osmolality, butdefinitive treatment requires eliminating the underlying cause, if possible. In the case of severe, acutehyponatremia with symptoms (e.g., confusion, obtundation, seizures), hypertonic (3%) saline can be slowlyinfused intravenously but can have dangerous neurologic side effects.

Elevated serum glucose levels give rise to a factitious hyponatremia, but not SIAD.

Ref: Ellison DH, Berl T: The syndrome of inappropriate antidiuresis. N Engl J Med 2007;356(2):2064-2072.

Item 216

ANSWER: E

This is a classic presentation for slipped capital femoral epiphysis (SCFE) in an adolescent male who hasprobably had a recent growth spurt. Pain with activity is the most common presenting symptom, as opposedto the nighttime pain that is typical of malignancy. Obese males are affected more often. The pain istypically in the anterior thigh, but in a high percentage of patients the pain may be referred to the knee, lowerleg, or foot. Limited internal rotation of the hip, especially with the hip in 90° flexion, is a reliable andspecific finding for SCFE and should be looked for in all adolescents with hip, thigh, or knee pain.

Meralgia paresthetica is pain in the thigh related to entrapment of the lateral femoral cutaneous nerve, oftenattributed to excessively tight clothing. Legg-Calvé-Perthes disease (avascular or aseptic necrosis of thefemoral head) is more likely to occur between the ages of 4 and 8 years. Juvenile rheumatoid arthritistypically is associated with other constitutional symptoms including stiffness, fever, and pain in at least oneother joint, with the pain not necessarily associated with activity.

Ref: Griffin LY (ed): Essentials of Musculoskeletal Care, ed 3. American Academy of Orthopaedic Surgeons, American Academyof Pediatrics, 2005, pp 941-943.

Item 217

ANSWER: A

Hemolytic anemia is established by reticulocytosis, increased unconjugated bilirubin, elevated lactatedehydrogenase (LDH), decreased haptoglobin, and peripheral blood smear findings.

Ref: Dhaliwal G, Cornett PA, Tierney LM Jr: Hemolytic anemia. Am Fam Physician 2004;69(11):2599-2606.

Item 218

ANSWER: B

Use of an incentive spirometer and similar lung expansion techniques such as chest physiotherapy have beenshown to have significant benefit for reducing postoperative complications. A course of preoperativecorticosteroids has been shown to be beneficial, such as in a patient with COPD. Prolonged antibioticprophylaxis and postoperative total parenteral nutrition have not been shown to have any benefit.

Ref: Smetana GW: A 68-year-old man with COPD contemplating colon cancer surgery. JAMA 2007;297(19):2121-2130.

Page 71: 2008 It e Critique

70

Item 219

ANSWER: E

It is estimated that 3%–10% of infants and toddlers refuse to eat according to their caregivers. Unlike otherfeeding problems such as colic, this problem tends to persist without intervention. It is recommended thatcaregivers establish food rules, such as healthy scheduled meals and snacks, and apply them consistently.Parents should control what, when, and where children are being fed, whereas children should control howmuch they eat at any given time in accordance with physiologic signals of hunger and fullness. No food ordrinks other than water should be offered between meals or snacks. Food should not be offered as a rewardor present. Parents can be reassured that a normal child will learn to eat enough to prevent starvation. Ifmalnutrition does occur, a search for a physical or mental abnormality should be sought.

Ref: Bernard-Bonnin AC: Feeding problems of infants and toddlers. Can Fam Physician 2006;52:1247-1251.

Item 220

ANSWER: C

Acute monoarthritis in adults is most commonly caused by infection, trauma, or crystal deposition.Rheumatoid arthritis infrequently presents as monoarthritis, and more often has a subacute course withmultiple, symmetric joints involved. Although osteoporosis may result in a fracture of the knee joint withouttrauma, there is no reason to believe that this patient has a torn anterior cruciate ligament. Gonococcalarthritis is one of the most common causes of septic arthritis, but is highly unlikely in this elderly, sexuallyinactive patient. Nongonococcal septic arthritis (especially due to staphylococcal and streptococcal bacteria)is still a consideration and should be ruled out by aspiration of fluid to be sent for culture.

This patient’s presentation is most consistent with pseudogout. Having a normal uric acid level suggestsagainst gout, but does not rule it out. Also, gout is seven times more likely to be seen in males, whereaspseudogout is 1.5 times more frequent in females. Pseudogout most often affects the elderly, and usuallyaffects the knee, wrist, and ankle. Gout presents most commonly in the first metatarsophalangeal joint andinsteps of the feet, but also can occur in the knee, wrist, finger, and olecranon bursa.

Differentiating between gout and pseudogout can be difficult and is best done by analysis of joint fluid. Inpatients with gout, this fluid contains highly negative birefringent, needle-shaped urate crystals, whereas inpseudogout the fluid contains rhomboid-shaped, weakly positive birefringent calcium pyrophosphate crystals.

Ref: Siva C, Velazquez C, Mody A, et al: Diagnosing acute monoarthritis in adults: A practical approach for the family physician.Am Fam Physician 2003;68(1):83-90. 2) Schlesinger N: Diagnosis of gout: Clinical, laboratory, and radiologic findings. AmJ Manag Care 2005;11(15 Suppl):S443-S450.

Item 221

ANSWER: E

End-of-life issues are a challenge to primary care physicians because of concerns about a lack of educationin pain control and palliative care. Trying to determine the prognosis of patients is difficult, and even withestablished criteria, the estimated prognosis is right only 50% of the time. There is a tendency for mostphysicians to overestimate life expectancy in a terminal patient.

Page 72: 2008 It e Critique

71

Most patients who are appropriate candidates for hospice care do not receive referrals until late in theirillness, if at all. Patients at the end of life have five main areas of concern: control of pain and othersymptoms; avoiding a prolongation of the dying process; having a sense of control; relieving burdens onfamily and loved ones; and strengthening relationships with family and friends.

Ref: Standridge JB, Zylstra RG, Miller KE, et al: Caring for Elderly Individuals. AAFP Home Study Self-Assessment monographseries, 2004, no 294, pp 53-55.

Item 222

ANSWER: B

Hand, foot, and mouth disease is a mild infection occurring in young children, and is caused bycoxsackievirus A16, or occasionally by other strains of coxsackie- or enterovirus. In addition to the orallesions, vesicular lesions may occur on the feet and nonvesicular lesions may occur on the buttocks. Alow-grade fever may also develop. Herpangina is also caused by coxsackieviruses, but it is a more severeillness characterized by severe sore throat and vesiculo-ulcerative lesions limited to the tonsillar pillars, softpalate, and uvula, and occasionally the posterior oropharynx. Temperatures can range to as high as 41°C(106°F). The etiology of aphthous stomatitis is multifactorial, and it may be due to a number of conditions.Systemic signs, such as fever, are generally absent. Lesions are randomly distributed. Herpeticgingivostomatitis also causes randomly distributed oral ulcers, but it is a more severe illness, regularlyaccompanied by a higher fever, and is extremely painful. Streptococcal pharyngitis is rarely accompaniedby ulceration except in agranulocytic patients.

Ref: Kliegman RM, Behrman RE, Jenson HB, et al (eds): Nelson Textbook of Pediatrics, ed 18. Saunders, 2007, pp 1352,1361-1362, 1752-1753, 2735.

Item 223

ANSWER: B

Omega-3 fats contribute to the production of eicosapentaenoic acid (EPA) and docosahexaenoic acid (DHA),which inhibit the inflammatory immune response and platelet aggregation, are mild vasodilators, and mayhave antiarrhythmic properties. The American Heart Association guidelines state that omega-3 supplementsmay be recommended to patients with preexisting disease, a high risk of disease, or high triglyceride levels,as well as to patients who do not like or are allergic to fish. The Italian GISSI study found that the use of850 mg of EPA and DHA daily resulted in decreased rates of mortality, nonfatal myocardial infarction, andstroke, with particular decreases in the rate of sudden death.

Ref: Olendzki B, Speed C, Domino FJ: Nutritional assessment and counseling for prevention and treatment of cardiovasculardisease. Am Fam Physician 2006;73(2):257-264.

Item 224

ANSWER: E

Late decelerations are thought to be associated with uteroplacental insufficiency and fetal hypoxia due todecreased blood flow in the placenta. This pattern is a warning sign and is associated with increasing fetalcompromise, worsening fetal acidosis, fetal central nervous system depression, and/or direct myocardialhypoxia. Early decelerations are thought to result from vagus nerve response to fetal head compression, andare not associated with increased fetal mortality or morbidity. Variable decelerations are thought to be dueto acute, intermittent compression of the umbilical cord between fetal parts and the contracting uterus.

Page 73: 2008 It e Critique

72

Ref: Cunningham FG, Leveno KJ, Bloom SL, et al: Williams Obstetrics, ed 22. McGraw-Hill, 2005, pp 452-453.

Item 225

ANSWER: A

There should be a rational approach to evaluating weight loss in an elderly patient. The workup should bedirected by findings in the history and physical examination, with special emphasis given to neurologic andpsychosocial aspects. Unless the history or physical examination point in a specific direction, standard testsshould be performed first, including a CBC, chemistry panel, stool for occult blood, and TSH level.Although the etiology of unintentional weight loss in the elderly is malignancy in 16%–36% of such cases,specific tests are not indicated before CT.

Medications, including SSRIs, NSAIDs, bupropion, digoxin, and metformin can cause weight loss; however,amitriptyline often leads to weight gain. Mirtazapine has been shown to increase appetite and promoteweight gain. Megestrol has been used successfully to treat cachexia in patients with AIDS or cancer. Whengiven in a dosage of at least 320 mg/day, megestrol has produced weight gain, but side effects of edema,constipation, and delirium may limit its usefulness. Lower dosages may be effective for stimulating weightgain in frail elderly patients, although this approach needs to be tested in randomized, controlled trials. Inthe patient described, a workup seeking the etiology of the weight loss should begin promptly.

Ref: Bouras EP, Lange SM, Scolapio JS: Rational approach to patients with unintentional weight loss. Mayo Clin Proc2001;76(9):923-929. 2) Huffman GB: Evaluating and treating unintentional weight loss in the elderly. Am Fam Physician2002;65(4):640-650. 3) Alibhai SM, Greenwood C, Payette H: An approach to the management of unintentional weight lossin elderly people. CMAJ 2005;172(6):773-780.

Item 226

ANSWER: A

This patient has prediabetes, which is defined as having a fasting plasma glucose level of 101–125 mg/dL.These patients are at high risk for developing diabetes mellitus later in life. Prediabetes is associated withmetabolic syndrome, and weight loss, exercise, and certain pharmacologic agents have been shown to preventor delay the subsequent development of diabetes mellitus.

Diabetes mellitus is diagnosed in three ways: symptoms of diabetes (polyuria, polydipsia, unexplainedweight loss) plus a random plasma glucose level �200 mg/dL; a fasting plasma glucose level �126 mg/dL;or a glucose level �200 mg/dL on a 2-hour 75-g oral glucose tolerance test. It is important to note that in theabsence of unequivocal hyperglycemia the diagnosis must be confirmed by repeat testing on a subsequentday. Once the diagnosis of diabetes is confirmed, further testing is needed to differentiate between type 1,type 2, and maturity-onset diabetes of youth.

Ref: American Diabetes Association: Diagnosis and classification of diabetes mellitus. Diabetes Care 2008;31(Suppl 1):S55-S60.2) Peterson K, Silverstein J, Kaufman F, et al: Management of type 2 diabetes in youth: An update. Am Fam Physician2007;76(5):658-664.

Page 74: 2008 It e Critique

73

Item 227

ANSWER: E

The U.S. Preventive Services Task Force states that there is insufficient evidence to recommend either foror against screening for cancer of the lung. To date, screening has not been shown to decrease the numberof deaths from lung cancer. Case control studies done in Japan suggest improved mortality with annual chestradiographs, and a large randomized, controlled trial is now under way. While screening CT in high-riskgroups would identify a high percentage of stage 1 lung cancers, there is no data available at this time fromrandomized studies to show that this is worthwhile. Studies of this issue are also currently under way,however.

Ref: Deffebach ME, Humphrey L: Screening for lung cancer. UpToDate 2006;version 14.3. 2) Bach PB, Jett JR, Pastorino U,et al: Computed tomography screening and lung cancer outcomes. JAMA 2007;297(9):953-961.

Item 228

ANSWER: C

Rotavirus vaccine (RotaTeq) was licensed in February 2006 to protect against viral gastroenteritis. TheAdvisory Committee on Immunization Practices recommends the routine vaccination of infants with threedoses to be given at 2, 4, and 6 months of age. The first dose should be given between 6 and 12 weeks ofage, and subsequent doses should be given at 4- to 10-week intervals, but all three doses should beadministered by 32 weeks of age. Unlike the vaccine RotaShield, which was marketed in 1999, RotaTeq isnot known to increase the risk for intussusception.

Ref: Parashar UD, Alexander JP, Glass RI, et al: Prevention of rotavirus gastroenteritis among infants and children:Recommendations of the Advisory Committee on Immunization Practices. MMWR 2006;55(RR-12):1-13.

Item 229

ANSWER: A

Croup can be classified as spasmodic croup, laryngotracheitis, laryngotracheobronchitis (LTB),laryngotracheobroncheopneumonia (LTBP), or laryngeal diphtheria. Mild croup is manifested by anoccasional barking cough with no stridor at rest, and mild or absent intercostal retractions. Moderate crouppresents with a more frequent barking cough, stridor with suprasternal and sternal retractions at rest, but noagitation. Severe croup includes more prominent inspiratory and expiratory stridor with agitation anddistress.

There is good evidence that corticosteroids produce significant improvement. The regimens studied mostfrequently have consisted of single-dose dexamethasone (0.6 mg/kg orally or intramuscularly), with somestudies including up to four more doses over a 2-day period. Longer courses of corticosteroids have notproven to be more effective and may be harmful, leading to secondary infections. Racemic epinephrine bynebulization is indicated in severe croup. Antitussives and decongestants have not been studied and are notrecommended. Antibiotics are indicated in LTB and LTBP, which can be diagnosed on the basis of cracklesand wheezing on examination, or by an abnormal chest radiograph. Laryngotracheitis can sometimes beassociated with a bacterial infection, but should be suspected only after a patient does not improve withcorticosteroids and epinephrine.

Ref: Cherry JD: Croup. N Engl J Med 2008;358(4):384-391.

Page 75: 2008 It e Critique

74

Item 230

ANSWER: C

12Patients with renal failure often have normal vitamin B levels despite an actual deficiency. In this situation,

12the clinician can order a methylmalonic acid (MMA) level to confirm the diagnosis. Vitamin B is the

12necessary coenzyme in the metabolism of MMA to succinylcholine. Thus, in the absence of vitamin B ,

12MMA levels increase. Additionally, homocysteine levels would be elevated in the presence of vitamin Bdeficiency (SOR A).

12Ref: Cravens DD, Nashelsky J: How to we evaluate a marginally low B level? J Fam Pract 2007;56(1):62-63.

Item 231

ANSWER: A

The American College of Rheumatology recommends that patients who are beginning long-term treatmentwith prednisone (�3 months at a dosage �5 mg/day), or an equivalent, receive bisphosphonate therapy inaddition to calcium and vitamin D supplementation, regardless of their T score. The other treatments are notrecommended for prevention of glucocorticoid-induced osteoporosis.

Ref: Zizic TM: Pharmacologic prevention of osteoporotic fractures. Am Fam Physician 2004;70(7):1293-1300.

Item 232

ANSWER: C

This EKG tracing is most consistent with acute inferior myocardial infarction with ST-T–wave elevationsin inferior leads II, III, and aVF. The ST-T–segment changes in lead II appear more nonspecificallyabnormal, but with clear elevations of the ST-T segments in leads III and aVF, acute myocardial infarctionis most likely. This can be corroborated by the so-called reciprocal ST-T–segment depressions in leads I andaVL. The ST-T–segment depressions in leads I, aVL, and to a lesser extent V6, might be seen in high lateralwall ischemia, but the inferior leads would not exhibit the ST-T elevations if this were just ischemia.

Although the QRS complex is slightly widened in this tracing (107 msec), right bundle branch block is notpresent. With P waves before each QRS, the rhythm is sinus, not atrial flutter. None of the findings in thisEKG are characteristic of pulmonary embolism.

Ref: Fauci AS, Braunwald E, Kasper DL, et al (eds): Harrison's Principles of Internal Medicine, ed 17. McGraw-Hill, 2008, pp1391-1396.

Page 76: 2008 It e Critique

75

Item 233

ANSWER: B

Although the consequences of nasal (or facial) staphylococcal lesions demand that they receive carefulevaluation and culture, most lesions in this area of the face are related to HSV-1 infections. The painfulgrouped vesicles on an erythematous base indicate a viral infection, likely HSV-1. Herpes zoster would beunlikely because the lesion involves only the distal nose area without other lesions, and symptoms would beunlikely. The appearance of staphylococcal infection would be quite different, contact dermatitis would bemore widespread, and orf virus is a sheep-related virus usually seen on the hands.

Ref: Habif TP, Campbell JL Jr, Chapman MS, et al: Skin Disease: Diagnosis and Treatment, ed. 2 Elsevier, 2005, pp 198-203.

Item 234

ANSWER: C

The patient’s symptoms and anoscopic examination are consistent with the diagnosis of acute proctitis.Possible causes include Chlamydia, gonorrhea, and ulcerative proctitis. Parasitic diseases such asEntamoeba histolytica can cause dysentery, but Giardia lamblia causes nausea, vomiting, malabsorption,and watery diarrhea rather than bloody diarrhea, and is an infection of the duodenum and proximal jejunum.Clostridium difficile infection is also not limited to the rectum, and the mucosa has a whitish, membranousappearance rather than an ulcerative appearance. The anoscopic examination did not reveal a nodular masstypical of colorectal cancer.

Ref: Kucik CJ, Martin GL, Sortor BV: Common intestinal parasites. Am Fam Physician 2004;69(5):1161-1168.

Item 235

ANSWER: D

The clinical picture of diverticulitis can be confused with urinary tract infections because bladder symptomscan accompany the gastrointestinal symptoms. The sigmoid colon lies close enough to the bladder that thebladder can be irritated by sigmoid diverticulitis. Conversely, urinary infections do not cause painfuldefecation. The location of the tenderness in the left lower quadrant and above the rectum is typical forsigmoid diverticulitis. No single test is routinely diagnostic for diverticulitis, but a CT scan that showsedema in the sigmoid mesentery has the best positive predictive value.

Appendicitis is usually diagnosed clinically, but like diverticulitis is increasingly confirmed by CT. Thetenderness is generally in the right lower quadrant. CT findings are noted around the cecum, rather thanadjacent to the sigmoid. Colon cancer is usually asymptomatic when discovered, but when symptoms arepresent they usually include a change in bowel habits, hematochezia, anemia, or symptoms of obstruction.Pericolic abscess is a rare presentation of colon cancer.

Ref: Rudy DR, Zdon MJ: Update on colorectal cancer. Am Fam Physician 2000;61(6):1759-1770. 2) Salzman H, Lillie D:Diverticular disease: Diagnosis and treatment. Am Fam Physician 2005;72(7):1229-1234.

Page 77: 2008 It e Critique

76

Item 236

ANSWER: D

The second EKG includes ST depression and T-wave inversion in leads V2–V4, the classic picture ofanterior ischemia. The patient did have a tight stenosis of his proximal left anterior descending artery. Thefirst EKG, from just half an hour earlier, removes any doubt that these features are due to chronic conditionssuch as left ventricular hypertrophy with strain or early repolarization. Although there is considerablebaseline artifact in the first EKG, it is still interpretable, especially with the additional information from thesecond EKG.

Ref: Achar SA, Kundu S, Norcross WA: Diagnosis of acute coronary syndrome. Am Fam Physician 2005;72(1):119-126.

Item 237

ANSWER: C

Recognizing common fracture types is an important part of determining how to proceed when caring for aninjured patient. Fractures in children can be different from those in adults for several reasons, including theelasticity of immature bone, the possibility of child abuse, and the presence of growth plates. The radiographshown with this question is an example of a fracture through the growth plate. Approximately 6%–7% ofsuch fractures will cause a restriction of growth. The Salter classification system was developed to classifyfractures into the growth plate and can be used to estimate the risk of growth restriction. The higher theclassification, the greater the risk of complications.

Ref: Rudolph CD, Rudolph AM (eds): Rudolph’s Pediatrics, ed 21. McGraw-Hill, 2003, pp 2450-2451.

Item 238

ANSWER: B

The lesions shown are typical of wheals associated with urticaria. Acute urticaria has a variety of causes,but often is idiopathic. The lesions are sharply defined and can range from <1 cm to >8 cm. The color iserythematous, or white with an erythematous rim, and lesions can be round, oval, acriform, annular (as withthis patient), or serpiginous. The lesions are often transient, fluctuating from hour to hour, and sometimesare associated with angioedema (edematous areas of dermis and subcutaneous tissue). Appropriate treatmentfor mildly symptomatic cases begins with antihistamines. Oral or parenteral corticosteroids would beappropriate if angioedema were significant (i.e., swelling of the face, mouth, or larynx).

Ref: Habif TP, Campbell JL Jr, Chapman MS, et al: Skin Disease: Diagnosis and Treatment, ed 2. Elsevier, 2005, pp 76-77.

Item 239

ANSWER: D

The clinical history and the examination suggest the possibility of a left ventricular (LV) aneurysm. TheEKG demonstrates an anteroseptal infarction pattern with persistent ST-segment elevation. When STelevation persists for 3 or more weeks following myocardial infarction an LV aneurysm should beconsidered. LV aneurysms often lead to symptoms of heart failure, severe angina, ventricular arrhythmias,and mural thrombosis. Thrombus formation sometimes results in systemic embolization. Two dimensionalechocardiography, radionuclide ventriculography, and cardiac MRI are useful to confirm the diagnosis.

Page 78: 2008 It e Critique

77

Pericarditis is associated with ST elevation, but in a diffuse pattern with J-point elevation and a concavemorphology of the ST segment. A ruptured papillary muscle would result in acute heart failure with amurmur of mitral regurgitation within the first few weeks following a myocardial infarction. Nocharacteristic EKG abnormalities are found with this complication. Pulmonary embolism and left ventricularhypertrophy are sometimes associated with EKG abnormalities, but not ST elevation, and have a differentclinical presentation.

Ref: Libby P, Bonow RO, Mann DL, et al (eds): Braunwald’s Heart Disease: A Textbook of Cardiovascular Medicine, ed 8.Saunders, 2008, p 1399.

Item 240

ANSWER: C

As with pulmonary tuberculosis, the symptoms of lung abscess are most often insidious and prolonged, butwhen coupled with the development of putrid sputum, the diagnosis of lung abscess seems much more likely.In this case, the chest radiograph confirms the diagnosis by demonstrating a partially fluid-filled cavitarylesion. Abscesses of the lung most commonly result from aspiration of oral bacteria, including anaerobes,which is much more likely with coexisting periodontal disease and the potential for aspiration, such as mightoccur with seizures or hypoglycemic episodes.

When selecting an initial antibiotic regimen, the physician should consider the probability of a polymicrobialinfection with oral flora, including anaerobes. Of the choices provided, only clindamycin will provideanaerobic coverage. Treatment length varies, but often a course of 4–6 weeks is required. Surgicalintervention is rarely required.

Ref: Kasper DL, Braunwald E, Fauci AS, et al (eds): Harrison’s Principles of Internal Medicine, ed 16. McGraw-Hill, 2005, pp942-943, 945.